Универсальный внешний накопитель для всех iOS-устройств, совместим с PC/Mac, Android
Header Banner
8 800 100 5771 | +7 495 540 4266
c 9:00 до 24:00 пн-пт | c 10:00 до 18:00 сб
0 Comments

Содержание

Явление самоиндукции.Индуктивность. Энергия магнитного поля тока. Работа поля. Тесты, курсы по физике

Тестирование онлайн

  • Явление самоиндукции. Индуктивность. Основные понятия

  • Явление самоиндукции. Энергия магнитного поля

Явление самоиндукции

Мы уже изучили, что около проводника с током возникает магнитное поле. А также изучили, что переменное магнитное поле порождает ток (явление электромагнитной индукции). Рассмотрим электрическую цепь. При изменении силы тока в этой цепи произойдет изменение магнитного поля, в результате чего в этой же цепи возникнет дополнительный индукционный ток. Такое явление называется самоиндукцией, а ток, возникающий при этом, называется током самоиндукции.

Явление самоиндукции – это возникновение в проводящем контуре ЭДС, создаваемой вследствие изменения силы тока в самом контуре.

Индуктивность контура зависит от его формы и размеров, от магнитных свойств окружающей среды и не зависит от силы тока в контуре.

ЭДС самоиндукции определяется по формуле:

Явление самоиндукции подобно явлению инерции. Так же, как в механике нельзя мгновенно остановить движущееся тело, так и ток не может мгновенно приобрести определенное значение за счет явления самоиндукции. Если в цепь, состоящую из двух параллельно подключенных к источнику тока одинаковых ламп, последовательно со второй лампой включить катушку, то при замыкании цепи первая лампа загорается практически сразу, а вторая с заметным запаздыванием.

При размыкании цепи сила тока быстро уменьшается, и возникающая ЭДС самоиндукции препятствует уменьшению магнитного потока. При этом индуцированный ток направлен так же, как и исходный. ЭДС самоиндукции может во многом раз превысить внешнюю ЭДС. Поэтому электрические лампочки очень часто перегорают при выключении света.

Энергия магнитного поля

Энергия магнитного поля контура с током:

Тест по физике на тему “Электромагнитная индукция”

Тест 11-1(электромагнитная индукция)

Вариант 1

1. Кто открыл явление электромагнитной индукции?

А. X. Эрстед. Б. Ш. Кулон. В. А. Вольта. Г. А. Ампер. Д. М. Фарадей. Е

. Д. Максвелл.

2. Выводы катушки из медного провода присоединены к чувствительному гальванометру. В каком из перечисленных опытов гальванометр обнаружит возникновение ЭДС электромагнитной индукции в катушке?

  1. В катушку вставляется постоянный магнит.

  2. Из катушки вынимается постоянный магнит.

  3. Постоянный магнит вращается вокруг своей продоль­ной оси внутри катушки.

А. Только в случае 1. Б. Только в случае 2. В. Только в случае 3. Г. В случаях 1 и 2. Д. В случаях 1, 2 и 3.

3.Как называется физическая величина, равная произве­дению модуля В индукции магнитного поля на площадь S поверхности, пронизываемой магнитным полем, и косинус
угла а между вектором В индукции и нормалью п к этой поверхности?

А. Индуктивность. Б. Магнитный поток. В. Магнитная индукция. Г. Са­моиндукция. Д. Энергия магнитного поля.

4. Каким из приведенных ниже выражений определяется ЭДС индукции в замкнутом контуре?

A. Б. В. Г. Д.

5. При вдвигании полосового магнита в металлическое кольцо и выдвигании из него в кольце возникает индук­ционный ток. Этот ток создает магнитное поле. Каким по­люсом обращено магнитное поле тока в кольце к: 1) вдвигаемому северному полюсу магнита и 2) выдвигаемому се­верному полюсу магнита.

A. 1 — северным, 2 — северным. Б. 1 — южным, 2 — южным.

B. 1 — южным, 2 — северным. Г. 1 — северным, 2 — южным.

6. Как называется единица измерения магнитного потока?

А. Тесла. Б. Вебер. В. Гаусс. Г. Фарад. Д. Генри.

7. Единицей измерения какой физической величины является 1 Генри?

А. Индукции магнитного ноля. Б. Электроемкости. В. Самоиндук­ции. Г. Магнитного потока. Д. Индуктивности.

8. Каким выражением определяется связь магнитного по­ тока через контур с индуктивностью L

контура и силой тока I в контуре?

A. LI. Б. . В. LI . Г. LI2. Д. .

9. Каким выражением определяется связь ЭДС самоин­дукции с силой тока в катушке?

А. Б. В. LI. Г. . Д. LI .

10. Ниже перечислены свойства различных полей. Какими из них обладает электростатическое поле?

  1. Линии напряженности обязательно связаны с электри­ческими зарядами.

  2. Линии напрялсенности не связаны с электрическими зарядами.

  3. Поле обладает энергией.

  4. Поле не обладает энергией.

  5. Работа сил по перемещению электрического заряда по замкнутому пути может быть не равна нулю.

  6. Работа сил по перемещению электрического заряда по любому замкнутому пути равна нулю.

А. 1, 4, 6. Б. 1, 3, 5. В. 1, 3, 6. Г. 2, 3, 5. Д. 2, 3, 6. Е. 2, 4, 6.

11. Контур площадью 1000 см2 находится в однородном магнитном поле с индукцией 0,5 Тл, угол между вектором В индукции и нормалью к поверхности контура 60°. Ка­ков магнитный поток через контур?

А. 250 Вб. Б. 1000 Вб. В. 0,1 Вб. Г. 2,5 · 10-2 Вб. Д. 2,5 Вб.

12. Какая сила тока в контуре индуктивностью 5 мГн создает магнитный поток 2 · 10-2 Вб?

А. 4 мА. Б. 4 А. В. 250 А. Г. 250 мА. Д. 0,1 А. Е. 0,1 мА.

13. Магнитный поток через контур за 5 · 10-2 с равномер­но уменьшился от 10 мВб до 0 мВб. Каково значение ЭДС в контуре в это время?

А. 5 · 10-4 В. Б. 0,1 В. В. 0,2 В. Г. 0,4 В. Д. 1 В. Е. 2 В.

14. Каково значение энергии магнитного поля катушки индуктивностью 5 Гн при силе тока в ней 400 мА?

А. 2 Дж. Б. 1 Дж. В. 0,8 Дж. Г. 0,4 Дж. Д. 1000 Дж. Е. 4·105 Дж.

15. Катушка, содержащая n витков провода, подключена к источнику постоянного тока с напряжением U на выходе. Каково максимальное значение ЭДС самоиндукции в катушке при увеличении напряжения на ее концах от 0 В до U В?

A, U В, Б. nU В. В. U/п В. Г. Может быть во много раз больше U, зависит от скорости изменения силы тока и от индуктивности катушки.

16. Две одинаковые лампы включены в цепь источника постоянного тока, первая последовательно с резистором, вторая последовательно с катушкой. В какой из ламп (рис. 1) сила тока при замыкании ключа К достигнет мак­симального значения позже другой?

А. В первой. Б. Во второй. В. В первой и второй одновременно. Г. В пер­вой, если сопротивление резистора больше сопротивления катушки. Д. Во второй, если сопротивление катушки больше сопротивления резистора.

17. Катушка индуктивностью 2 Гн включена параллельно с резистором электрическим сопротивлением 900 Ом, сила тока в катушке 0,5 А, электрическое сопротивление ка­тушки 100 Ом. Какой электрический заряд протечет в цепи катушки и резистора при отключении их от источника тока (рис. 2)?

А. 4000 Кл. Б. 1000 Кл. В. 250 Кл. Г. 1 • 10 -2 Кл. Д. 1,1 • 10-3 Кл. Е. 1 • 10-3 Кл.

18. Самолет летит со скоростью 900 км/ч, модуль вертикальной составляющей вектора индукции магнитного поля Земли 4 • 105 Тл. Какова разность потенциалов между концами крыльев самолета, если размах крыльев равен 50 м?

А. 1,8 В. Б. 0,9 В. В. 0,5 В. Г. 0,25 В.

19. Какой должна быть сила тока в обмотке якоря электромотора для того, чтобы на участок обмотки из 20 витков длиной 10 см, расположенный перпендикулярно век­тору индукции в магнитном поле с индукцией 1,5 Тл, действовала сила 120 Н?

А. 90 А. Б. 40 А. В. 0,9 А. Г. 0,4 А.

20. Какую силу нужно приложить к металлической пере­мычке для равномерного ее перемещения со скоростью 8 м/с по двум параллельным проводникам, располо­женным на расстоянии 25 см друг от друга в однородном магнитном поле с индукцией 2 Тл? Вектор индукции перпендикулярен плоскости, в которой расположены рельсы. Проводники замкнуты резистором с электрическим сопротивлением 2 Ом.

А. 10000 Н. Б. 400 Н. В. 200 Н. Г. 4 Н. Д. 2 Н. Е. 1 Н.

Вариант 2

1. Как называется явление возникновения электрического тока в замкнутом контуре при изменении магнитного по­тока через контур?

А. Электростатическая индукция. Б. Явление намагничивания. В. Сила Ампера. Г. Сила Лоренца. Д. Электролиз. Е. Электромагнитная индукция.

2. Выводы катушки из медного провода присоединены к чувствительному гальванометру. В каком из перечислен­ных опытов гальванометр обнаружит возникновение ЭДС электромагнитной индукции в катушке?

  1. В катушку вставляется постоянный магнит.

  2. Катушка надевается на магнит.

3)Катушка вращается вокруг магнита, находящегося
внутри нее.

А.В случаях 1, 2 и 3. Б. В случаях 1 и 2. В. Только в случае 1. Г. Только в случае 2. Д. Только в случае 3.

3. Каким из приведенных ниже выражений определяется магнитный поток?

A. BScosα. Б. . В. qvBsinα. Г. qvBI. Д. IBlsina.

4. Что выражает следующее утверждение: ЭДС индукции в замкнутом контуре пропорциональна скорости измене­ния магнитного потока через поверхность, ограниченную контуром?

А. Закон электромагнитной индукции. Б. Правило Ленца. В. Закон Ома для полной цепи. Г. Явление самоиндукции. Д. Закон электролиза.

5. При вдвигании полосового магнита в металлическое кольцо и выдвигании из него в кольце возникает индук­ционный ток. Этот ток создает магнитное поле. Каким полюсом обращено магнитное поле тока в кольце к: 1) вдви­гаемому южному полюсу магнита и 2) выдвигаемому южному полюсу магнита.

A. 1 — северным, 2 — северным. Б. 1 — южным, 2 — южным.

B. 1 — южным, 2 — северным. Г. 1 — северным, 2 — южным.

6. Единицей измерения какой физической величины является 1 Вебер?

А. Индукции магнитного поля. Б. Электроемкости. В. Самоиндукции. Г. Магнитного потока. Д. Индуктивности.

7. Как называется единица измерения индуктивности?

А. Тесла. Б. Вебер. В. Гаусс. Г. Фарад. Д. Генри.

8. Каким выражением определяется связь энергии маг­нитного потока в контуре с индуктивностью L контура и силой тока I в контуре?

А. . Б. . В. LI2

, Г. LI . Д. LI.

9.Какая физическая величина х определяется выражением х= для катушки из п витков.

А. ЭДС индукции. Б. Магнитный поток. В. Индуктивность. Г. ЭДС само­индукции. Д. Энергия магнитного поля. Е. Магнитная индукция.

10. Ниже перечислены свойства различных полей. Какими из них обладает вихревое индукционное электрическое поле?

  1. Линии напряженности обязательно связаны с электри­ческими зарядами.

  2. Линии напряженности не связаны с электрическими зарядами.

  3. Поле обладает энергией.

  4. Поле не обладает энергией.

  5. Работа сил по перемещению электрического заряда по замкнутому пути может быть не равна нулю.

  6. Работа сил по перемещению электрического заряда по любому замкнутому пути равна нулю.

А. 1, 4, 6. Б. 1, 3, 5. В. 1, 3, в. Г. 2, 3, 5. Д. 2, 3, 6. Е. 2, 4, 6.

11. Контур площадью 200 см2 находится в однородном магнитном поле с индукцией 0,5 Тл, угол между вектором В индукции и нормалью к поверхности контура 60°. Ка­ков магнитный поток через контур?

А. 50 Вб. Б. 2 · 10-2 Вб. В. 5 · 10-3 Вб. Г. 200 Вб. Д. 5 Вб.

12. Ток 4 А создает в контуре магнитный поток 20 мВб. Какова индуктивность контура?

А. 5 Гн. Б. 5 мГн. В. 80 Гн. Г. 80 мГн. Д. 0,2 Гн. Е. 200 Гн.

13. Магнитный поток через контур за 0,5 с равномерно уменьшился от 10 мВб до 0 мВб. Каково значение ЭДС в контуре в это время?

А. 5 · 10-3 В. Б. 5 В. В. 10 В. Г. 20 В. Д. 0,02 В. Е. 0,01 В.

14. Каково значение энергии магнитного поля катушки индуктивностью 500 мГн при силе тока в ней 4 А?

А. 2 Дж. Б. 1 Дж. В. 8 Дж. Г. 4 Дж. Д. 1000 Дж. Е. 4000 Дж.

15. Катушка, содержащая п витков провода, подключена к источнику постоянного тока с напряжением U на выхо­де. Каково максимальное значение ЭДС самоиндукции в катушке при уменьшении напряжения на ее концах от U В до 0 В?

A. U В. Б. nU В. В. U/n В. Г. Может быть во много раз больше U, зависит от скорости изменения силы тока и от индуктивности катушки.

16. В электрической цепи, представленной на рисунке 1, четыре ключа 1, 2, 3 и 4 замкнуты. Размыкание какого из четырех даст лучшую возможность обнаружить явление самоиндукции?

А. 1. Б. 2. В. 3. Г. 4. Д. Любого из четырех.

17. Катушка индуктивностью 2 Гн включена параллельно с резистором электрическим сопротивлением 100 Ом, сила тока в катушке 0,5 А, электрическое сопротивление ка­тушки 900 Ом. Какой электрический заряд протечет в це­пи катушки и резистора при отключении их от источника тока (рис. 2)?

А. 4000 Кл. Б. 1000 Кл. В. 250 Кл. Г. 1 • 10-2 Кл. Д. 1,1 • 10-3 Кл. Е. 1 • 10-3 Кл.

18. Самолет летит со скоростью 1800 км/ч, модуль вертикальной составляющей вектора индукции магнитного поля Земли 4 • 10-5 Тл. Какова разность потенциалов между кон­цами крыльев самолета, если размах крыльев равен 25 м?

А. 1,8 В. В. 0,5 В. В. 0,9 В. Г. 0,25 В.

19. Прямоугольная рамка площадью S с током I помеще­на в магнитном поле с индукцией В . Чему равен момент силы, действующей на рамку, если угол между вектором В и нормалью к рамке равен а?

A. IBS sin а. Б. IBS. В. IBS cos а. Г. I2BS sin а. Д. I2BS cos а.

20. По двум вертикальным рельсам, верхние концы кото­рых замкнуты резистором электрическим сопротивлением R, начинает скользить проводящая перемычка массой т и длиной I. Система находится в магнитном поле. Вектор индукции перпендикулярен плоскости, в которой расположены рельсы. Найдите установившуюся скорость и движения перемычки. Сила трения пренебрежимо мала.

А. . В. В. . Г. . Д. .

Ответы:

Номер вопроса и ответ

1

2

3

4

5

6

7

8

9

10

11

12

13

14

15

16

17

18

19

20

Вариант 1

Д

Г

Б

Б

Г

Б

Д

А

Д

В

Г

Б

В

Г

А

Б

Е

В

Б

Е

Вариант 2

Е

Б

А

А

В

Г

Д

Б

А

Г

В

Б

Д

Г

Г

А

Е

Б

В

А

Явление самоиндукции — урок. Физика, 9 класс.

Согласно правилу Ленца индукционный ток в замкнутом контуре всегда противодействует своим магнитным полем изменению внешнего магнитного потока, которое вызвало его появление.

Рассмотрим случай, когда явление электромагнитной индукции наблюдается при изменении силы тока, проходящего через катушку с большим количеством витков. Если причина возникновения индукционного тока состоит в возрастании тока, то индукционный ток своим магнитным полем будет противодействовать этому возрастанию.

Убедиться в этом можно на следующем опыте.

Соберем цепь по следующему принципу: один участок содержит электрическую лампу и катушку индуктивности, второй участок — электрическую лампу и сопротивление, которое одинаково с сопротивлением катушки. В замкнутом электрическом контуре на первом участке наблюдаем явление запаздывания включения лампы, которое называется самоиндукцией.

 

 

 

 

 

 

 

Самоиндукция — это явление возникновения индукционного тока в цепи при изменении протекающего по цепи тока.

Возникающий индукционный ток называют током самоиндукции.

На участке цепи, содержащий катушку, возник индукционный ток, который препятствовал нарастанию основного тока, создаваемого источником, поэтому лампа загорелась позже, чем лампа соединенная с резистором. Из этого следует, что индуктивность катушки превышает индуктивность резистора.

Индуктивность — это физическая величина, которую обозначают буквой L.

Индуктивность характеризует способность катушки препятствовать нарастанию силы тока.

Обрати внимание!

За единицу измерения индуктивности принят генри (Гн).

L=1 Гн

Различные катушки могут иметь разную индуктивность. Она зависит от:

  • размеров и формы катушки;
  • числа витков;
  • наличия сердечника;
  • материала, из которого изготовлен сердечник.

Чем большей индуктивностью обладает катушка, тем с большим запозданием будет загораться лампа.

Явление самоиндукции можно наблюдать и при размыкании цепи. Изменим цепь.

 

 

Параллельно источнику тока включены катушка и лампа.

В такой цепи наблюдается явление кратковременного свечения лампы при размыкании электрической цепи, что также объясняется правилом Ленца о явлении самоиндукции как механизме препятствия изменения тока в контуре.

Применение катушек с большими значениями индуктивности, которые являются одновременно технологическими элементами электрической цепи и источниками больших значений ЭДС самоиндукции, может приводить при разрывах цепи к электризации воздуха.

 

Самоиндукция. Индуктивность. Энергия магнитного поля тока

Самоиндукция. Индуктивность. Энергия магнитного поля тока

Подробности
Просмотров: 628

Самоиндукция

Каждый проводник, по которому протекает эл.ток, находится в собственном магнитном поле.

При изменении силы тока в проводнике меняется м.поле, т.е. изменяется магнитный поток, создаваемый этим током. Изменение магнитного потока ведет в возникновению вихревого эл.поля и в цепи появляется ЭДС индукции.

Это явление называется самоиндукцией.

Самоиндукция – явление возникновения ЭДС индукции в эл.цепи в результате изменения силы тока.
Возникающая при этом ЭДС называется ЭДС самоиндукции


Проявление явления самоиндукции

Замыкание цепи

При замыкании в эл.цепи нарастает ток, что вызывает в катушке увеличение магнитного потока, возникает вихревое эл.поле, направленное против тока, т.е. в катушке возникает ЭДС самоиндукции, препятствующая нарастанию тока в цепи (вихревое поле тормозит электроны).
В результате Л1 загорается позже, чем Л2.

Размыкание цепи

При размыкании эл.цепи ток убывает, возникает уменьшение м.потока в катушке, возникает вихревое эл.поле, направленное как ток (стремящееся сохранить прежнюю силу тока) , т.е. в катушке возникает ЭДС самоиндукции, поддерживающая ток в цепи.
В результате Л при выключении ярко вспыхивает.

Вывод:

в электротехнике явление самоиндукции проявляется при замыкании цепи (электрический ток нарастает постепенно) и при размыкании цепи (электрический ток пропадает не сразу).



ИНДУКТИВНОСТЬ

От чего зависит ЭДС самоиндукции?

Электрический ток создает собственное магнитное поле. Магнитный поток через контур пропорционален индукции магнитного поля (Ф ~ B), индукция пропорциональна силе тока в проводнике
(B ~ I), следовательно магнитный поток пропорционален силе тока (Ф ~ I).
ЭДС самоиндукции зависит от скорости изменения силы тока в эл.цепи, от свойств проводника (размеров и формы) и от относительной магнитной проницаемости среды, в которой находится проводник.
Физическая величина, показывающая зависимость ЭДС самоиндукции от размеров и формы проводника и от среды, в которой находится проводник, называется коэффициентом самоиндукции или индуктивностью.

Индуктивность – физическая величина, численно равная ЭДС самоиндукции, возникающей в контуре при изменении силы тока на 1Ампер за 1 секунду.
Также индуктивность можно рассчитать по формуле:

где Ф – магнитный поток через контур, I – сила тока в контуре.

Единицы измерения индуктивности в системе СИ:

Индуктивность катушки зависит от:
числа витков, размеров и формы катушки и от относительной магнитной проницаемости среды ( возможен сердечник).

ЭДС САМОИНДУКЦИИ

ЭДС самоиндукции препятствует нарастанию силы тока при включении цепи и убыванию силы тока при размыкании цепи.

ЭНЕРГИЯ МАГНИТНОГО ПОЛЯ ТОКА

Вокруг проводника с током существует магнитное поле, которое обладает энергией.
Откуда она берется? Источник тока, включенный в эл.цепь, обладает запасом энергии.
В момент замыкания эл.цепи источник тока расходует часть своей энергии на преодоление действия возникающей ЭДС самоиндукции. Эта часть энергии, называемая собственной энергией тока, и идет на образование магнитного поля.

Энергия магнитного поля равна собственной энергии тока.
Собственная энергия тока численно равна работе, которую должен совершить источник тока для преодоления ЭДС самоиндукции, чтобы создать ток в цепи.

Энергия магнитного поля, созданного током, прямо пропорциональна квадрату силы тока.
Куда пропадает энергия магнитного поля после прекращения тока? – выделяется ( при размыкании цепи с достаточно большой силой тока возможно возникновение искры или дуги)

ВОПРОСЫ К ПРОВЕРОЧНОЙ РАБОТЕ

по теме “Электромагнитная индукция”

1. Перечислить 6 способов получения индукционного тока.
2. Явление электромагнитной индукции (определение).
3. Правило Ленца.
4. Магнитный поток ( определение, чертеж, формула, входящие величины, их ед. измерения).
5. Закон электромагнитной индукции (определение, формула).
6. Свойства вихревого электрического поля.
7. ЭДС индукции проводника, движущегося в однородном магнитном поле ( причина появления, чертеж, формула, входящие величины, их ед. измерения).
8. Самоиндукция (кратко проявление в электротехнике, определение).
9. ЭДС самоиндукции (ее действие и формула).
10. Индуктивность (определение, формулы, ед. измерения).
11. Энергия магнитного поля тока (формула, откуда появляется энергия м. поля тока, куда пропадает при прекращении тока).


Электромагнитное поле – Класс!ная физика

Взаимодействие токов. Магнитное поле. Вектор магнитной индукции. Сила Ампера — Действие магнитного поля на движущийся заряд.Магнитные свойства вещества — Явление электромагнитной индукции. Магнитный поток. Направление индукционного тока. Правило Ленца — ЭДС электромагнитной индукции. Вихревое электрическое поле — ЭДС индукции в движущихся проводниках
— Самоиндукция. Индуктивность. Энергия магнитного поля. Вопросы к пр/работе

Эдс самоиндукции катушки определяется по формуле. Явление самоиндукции

Электрический ток, проходящий по проводнику, создает вокруг него магнитное поле. Магнитный поток Ф через контур из этого проводника пропорционален модулю индукции В магнитного поля внутри контура, а индукция магнитного поля в свою очередь пропорциональна силе тока в проводнике. Следовательно, магнитный поток через контур прямо пропорционален силе тока в контуре:

Коэффициент пропорциональности между силой тока I в контуре и магнитным потоком Ф, создаваемым этим током, называется индуктивностью. Индуктивность зависит от размеров и формы проводника, от магнитных свойств среды, в которой находится проводник.

Единица индуктивности.

За единицу индуктивности в Международной системе принимается генри Эта единица определяется на основании формулы (55.1):

Индуктивность контура равна если при силе постоянного тока 1 А магнитный поток через контур равен

Самоиндукция.

При изменении силы тока в катушке происходит изменение магнитного потока, создаваемого этим током. Изменение магнитного потока, пронизывающего катушку, должно вызывать появление ЭДС индукции в катушке. Явление возникновения ЭДС индукции в

электрической цепи в результате изменения силы тока в этой цепи называется самоиндукцией.

В соответствии с правилом Ленца ЭДС самоиндукции препятствует нарастанию силы тока при включении и убыванию силы тока при выключении цепи.

Явление самоиндукции можно наблюдать, собрав электрическую цепь из катушки с большой индуктивностью, резистора, двух одинаковых ламп накаливания и источника тока (рис. 197). Резистор должен иметь такое же электрическое сопротивление, как и провод катушки. Опыт показывает, что при замыкании цепи электрическая лггмпа, включенная последовательно с катушкой, загорается несколько позже, чем лампа, включенная последовательно с резистором. Нарастанию тока в цепи катушки при замыкании препятствует ЭДС самоиндукции, возникающая при возрастании магнитного потока в катушке. При отключении источника тока вспыхивают обе лампы. В этом случае ток в цепи поддерживается ЭДС самоиндукции, возникающей при убывании магнитного потока в катушке.

ЭДС самоиндукции возникающая в катушке с индуктивностью по закону электромагнитной индукции равна

ЭДС самоиндукции прямо пропорциональна индуктивности катушки и скорости изменения силы тока в катушке.

Используя выражение (55.3), можно дать второе определение единицы индуктивности: элемент электрической цепи обладает индуктивностью в если при равномерном изменении силы тока в цепи на 1 А за 1 с в нем возникает ЭДС самоиндукции 1 В.

Энергия магнитного поля.

При отключении катушки индуктивности от источника тока лампа накаливания, включенная параллельно катушке, дает кратковременную вспышку. Ток в цепи возникает под действием ЭДС самоиндукции. Источником энергии, выделяющейся при этом в электрической цепи, является магнитное поле катушки.

Энергию магнитного поля катушки индуктивности можно вычислить следующим способом. Для упрощения расчета рассмотрим такой случай, когда после отключения катушки от источника ток в цепи убывает со временем по линейному закону. В этом случае ЭДС самоиндукции имеет постоянное значение, равное

Изменяющийся по величине ток всегда создает изменяющееся магнитное поле, которое, в свою очередь, всегда индуктирует ЭДС. При всяком изменении тока в катушке (или вообще в проводнике) в ней самой индуктируется ЭДС самоиндукции. Когда ЭДС в катушке индуктируется за счет изменения собственного магнитного потока, величина этой ЭДС зависит от скорости изменения тока. Чем больше скорость изменения тока, тем больше ЭДС самоиндукции. Величина ЭДС самоиндукции зависит также от числа витков катушки, густоты их намотки и размеров катушки. Чем больше диаметр катушки, число ее витков и густота намотки, тем больше ЭДС самоиндукции. Эта зависимость ЭДС самоиндукции от скорости изменения тока в катушке, числа ее витков и размеров имеет большое значение в электротехнике. Направление ЭДС самоиндукции определяется по закону Ленца. ЭДС самоиндукции имеет всегда такое направление, при котором она препятствует изменению вызвавшего ее тока.

Дисперсия света (разложение света) – это явление, обусловленное зависимостью абсолютного показателя преломления вещества от частоты (или длины волны) света (частотная дисперсия), или, то же самое, зависимость фазовой скорости света в веществе от длины волны (или частоты). Экспериментально открыта Ньютоном около 1672 года, хотя теоретически достаточно хорошо объяснена значительно позднее.

Пространственной дисперсией называется зависимость тензора диэлектрической проницаемости среды от волнового вектора. Такая зависимость вызывает ряд явлений, называемых эффектами пространственной поляризации.

Один из самых наглядных примеров дисперсии – разложение белого света при прохождении его через призму (опыт Ньютона). Сущностью явления дисперсии является различие скоростей распространения лучей света c различной длиной волны в прозрачном веществе – оптической среде (тогда как в вакууме скорость света всегда одинакова, независимо от длины волны и следовательно цвета). Обычно, чем больше частота световой волны, тем больше показатель преломления среды для неё и тем меньше скорость волны в среде:

у света красного цвета скорость распространения в среде максимальна, а степень преломления – минимальна,

у света фиолетового цвета скорость распространения в среде минимальна, а степень преломления – максимальна.

Разложение белого света призмой в спектр было известно очень давно. Однако разобраться в этом явлении до Ньютона никто не смог.

Ученых, занимающихся оптикой, интересовал вопрос о природе цвета. Наиболее распространенным было мнение о том, что белый свет является простым. Цветные же лучи получаются в результате тех или иных его изменений. Существовали различные теории по этому вопросу, на которых мы останавливаться не будем.

Изучая явление разложения белого света в спектр, Ньютон пришел к заключению, что белый свет является сложным светом. Он представляет собой сумму простых цветных лучей.

Ньютон работал с простой установкой.-3 Дж

1. Строение ядра. Модель атома. Опыты Резерфорда.

2. Трансформатор. Устройство, принцип действия, применение.

3. при разрядки батареи состоящей из 20 параллельно включенных одинаковых конденсаторов ёмкостью 4 мкФ каждый,выделилось 10 Дж тепла. Определить.до какой разности потенциалов были заряжены конденсаторы.

Ответы на Билет№26

1) Атомное ядро- центральная часть атома, в которой сосредоточена основная его масса (более 99,9 %). Ядро заряжено положительно, заряд ядра определяет химический элемент, к которому относят атом. Размеры ядер различных атомов составляют несколько фемтометров, что в более чем в 10 тысяч раз меньше размеров самого атома.

Атомные ядра изучает ядерная физика.

Атомное ядро состоит из нуклонов – положительно заряженных протонов и нейтральных нейтронов, которые связаны между собой при помощи сильного взаимодействия. Протон и нейтрон обладают собственным моментом количества движения (спином), равным и связанным с ним магнитным моментом. Единственный атом, не содержащий нейтрон в ядре – лёгкий водород (протий).

Атомное ядро, рассматриваемое как класс частиц с определённым числом протонов и нейтронов, принято называть нуклидом.

Атом – частица вещества микроскопических размеров и массы, наименьшая часть химического элемента, являющаяся носителем его свойств.

Атом состоит из атомного ядра и электронов. Если число протонов в ядре совпадает с числом электронов, то атом в целом оказывается электрически нейтральным. В противном случае он обладает некоторым положительным или отрицательным зарядом и называется ионом. В некоторых случаях под атомами понимают только электронейтральные системы, в которых заряд ядра равен суммарному заряду электронов, тем самым противопоставляя их электрически заряженным ионам.

Ядро, несущее почти всю (более чем 99,9 %) массу атома, состоит из положительно заряженных протонов и незаряженных нейтронов, связанных между собой при помощи сильного взаимодействия. Атомы классифицируются по количеству протонов и нейтронов в ядре: число протонов Z соответствует порядковому номеру атома в периодической системе Менделеева и определяет его принадлежность к некоторому химическому элементу, а число нейтронов N – определённому изотопу этого элемента. Единственный атом, не содержащий нейтронов в ядре – лёгкий водород (протий). Число Z также определяет суммарный положительный электрический заряд (Ze) атомного ядра и число электронов в нейтральном атоме, задающее его размер.

Атомы различного вида в разных количествах, связанные межатомными связями, образуют молекулы.

При изменении тока в контуре меняется поток магнитной индукции через поверхность , ограниченную этим контуром, изменение потока магнитной индукции приводит к возбуждению ЭДС самоиндукции. Направление ЭДС оказывается таким, что при увеличении тока в цепи эдс препятствует возрастанию тока, а при уменьшении тока – убыванию.

Величина ЭДС пропорциональна скорости изменения силы тока I и индуктивности контура L :

.

За счёт явления самоиндукции в электрической цепи с источником ЭДС при замыкании цепи ток устанавливается не мгновенно, а через какое-то время. Аналогичные процессы происходят и при размыкании цепи , при этом величина ЭДС самоиндукции может значительно превышать ЭДС источника. Чаще всего в обычной жизни это используется в катушках зажигания автомобилей. Типичное напряжение самоиндукции при напряжении питающей батареи 12В составляет 7-25кВ.

Wikimedia Foundation . 2010 .

Смотреть что такое “ЭДС самоиндукции” в других словарях:

    эдс самоиндукции – — [Я.Н.Лугинский, М.С.Фези Жилинская, Ю.С.Кабиров. Англо русский словарь по электротехнике и электроэнергетике, Москва, 1999 г.] Тематики электротехника, основные понятия EN self induced emfFaraday voltageinductance voltageself induction… …

    Это явление возникновения ЭДС индукции в проводящем контуре при изменении протекающего через контур тока. При изменении тока в контуре пропорционально меняется и магнитный поток через поверхность, ограниченную этим контуром. Изменение… … Википедия

    – (от лат. inductio наведение, побуждение), величина, характеризующая магн. св ва электрич. цепи. Ток, текущий в проводящем контуре, создаёт в окружающем пр ве магн. поле, причём магнитный поток Ф, пронизывающий контур (сцепленный с ним), прямо… … Физическая энциклопедия

    реактивная мощность – Величина, равная при синусоидальных электрическом токе и электрическом напряжении произведению действующего значения напряжения на действующее значение тока и на синус сдвига фаз между напряжением и током двухполюсника. [ГОСТ Р 52002 2003]… … Справочник технического переводчика

    Раздел физики, охватывающий знания о статическом электричестве, электрических токах и магнитных явлениях. ЭЛЕКТРОСТАТИКА В электростатике рассматриваются явления, связанные с покоящимися электрическими зарядами. Наличие сил, действующих между… … Энциклопедия Кольера

    Электрический машина, не имеющая подвижных частей и преобразующая переменный ток одного напряжения в переменный ток другого напряжения. В простейшем случае состоит из магнитопровода (сердечника) и расположенных на нём двух обмоток первичной и… … Энциклопедический словарь

Самоиндукцией называется появление в проводнике электродвижущей силы (ЭДС), направленной в противоположную сторону относительно напряжения источника питания при протекании тока. При этом оно возникает в момент, когда сила тока в цепи изменяется. Изменяющийся электрической ток порождает изменяющееся магнитное поле, оно в свою очередь наводит ЭДС в проводнике.

Это похоже на формулировку закона электромагнитной индукции Фарадея, где сказано:

При прохождении магнитного потока через проводник, в последнем возникает ЭДС. Она пропорциональна скорости изменения магнитного потока (мат. производная по времени).

E=dФ/dt ,

Где E – ЭДС самоиндукции, измеряется в вольтах, Ф – магнитный поток, единица измерения – Вб (вебер, он же равен В/с)

Индуктивность

Мы уже сказали о том, что самоиндукция присуща индуктивным цепям, поэтому рассмотрим явление самоиндукции на примере катушки индуктивности.

Катушка индуктивности – это элемент, который представляет собой катушку из изолированного проводника. Для увеличения индуктивности увеличивают число витков или внутрь катушки помещают сердечник из магнитомягкого или другого материала.

Единица измерения индуктивности – Генри (Гн). Индуктивность характеризует то, насколько сильно проводник противодействует электрическому току. Так как вокруг каждого проводника, по которому протекает ток, образуется магнитное поле, и, если поместить проводник в переменное поле – в нем возникнет ток. В свою очередь магнитные поля каждого витка катушки складываются. Тогда вокруг катушки, по которой протекает ток, возникнет сильное магнитное поле. При изменении его силы в катушке будет изменяться и магнитный поток вокруг неё.

Согласно закону электромагнитной индукции Фарадея, если катушку будет пронизывать переменный магнитный поток, то в ней возникнет ток и ЭДС самоиндукции. Они будут препятствовать току, который протекал в индуктивности от источника питания к нагрузке. Их еще называют экстратоки ЭДС самоиндукции.

Формула ЭДС самоиндукции на индуктивности имеет вид:

То есть чем больше индуктивность, и чем больше и быстрее изменился ток – тем сильнее будет всплеск ЭДС.

При возрастании тока в катушке возникает ЭДС самоиндукции, которая направлена против напряжения источника питания, соответственно возрастание тока замедлится. То же самое происходит при убывании – самоиндукция приведет к появлению ЭДС, которое будет поддерживать ток в катушке в том же направлении, что и до этого. Отсюда следует, что напряжение на выводах катушки будет противоположным полярности источника питания.

На рисунке ниже вы видите, что при включении/отключении индуктивной цепи ток не резко возникает, а изменяется постепенно. Об этом говорят и законы коммутации.

Другое определение индуктивности звучит так: магнитный поток пропорционален току, но в его формуле индуктивность выступает в качестве коэффициента пропорциональности.

Трансформатор и взаимоиндукция

Если расположить две катушки в непосредственной близости, например, на одном сердечнике, то будет наблюдаться явление взаимоиндукции. Пропустим переменный ток по первой, тогда её переменный поток будет пронизывать витки второй и на её выводах появится ЭДС.

Это ЭДС будет зависеть от длины провода, соответственно количества витков, а также от величины магнитной проницаемости среды. Если их расположить просто около друг друга — ЭДС будет низким, а если взять сердечник из магнитомягкой стали – ЭДС будет значительно больше. Собственно, так и устроен трансформатор.

Интересно: такое взаимное влияние катушек друг на друга называют индуктивной связью.

Польза и вред

Если вам понятна теоретическая часть, стоит рассмотреть где применяется явление самоиндукции на практике. Рассмотрим на примерах того, что мы видим в быту и технике. Одно из полезнейших применений – это трансформатор, принцип его работы мы уже рассмотрели. Сейчас встречаются все реже, но ранее ежедневно использовались люминесцентные трубчатые лампы в светильниках. Принцип их работы основан на явлении самоиндукции. Её схемы вы можете увидеть ниже.

После подачи напряжения ток протекает по цепи: фаза — дроссель — спираль — стартер — спираль — ноль.

Или наоборот (фаза и ноль). После срабатывания стартера, его контакты размыкаются, тогда (катушка с большой индуктивностью) стремится поддержать ток в том же направлении, наводит ЭДС самоиндукции большой величины и происходит розжиг ламп.

Аналогично это явление применяется в цепи зажигания автомобиля или мотоцикла, которые работают на бензине. В них в разрыв между катушкой индуктивности и минусом (массой) устанавливают механический (прерыватель) или полупроводниковый ключ (транзистор в ЭБУ). Этот ключ в момент, когда в цилиндре должна образоваться искра для зажигания топлива, разрывает цепь питания катушки. Тогда энергия, запасенная в сердечнике катушки, вызывает рост ЭДС самоиндукции и напряжение на электроде свечи возрастает до тех пор, пока не наступит пробой искрового промежутка, или пока не сгорит катушка.

В блоках питания и аудиотехнике часто возникает необходимость убрать из сигнала лишние пульсации, шумы или частоты. Для этого используются фильтры разных конфигурации. Один из вариантов это LC, LR-фильтры. Благодаря препятствию роста тока и сопротивлению переменного тока, соответственно, возможно добиться поставленных целей.

Вред ЭДС самоиндукции приносит контактам выключателей, рубильников, розеток, автоматов и прочего. Вы могли заметить что, когда вытаскиваете вилку работающего пылесоса из розетки, очень часто заметна вспышка внутри неё. Это и есть сопротивление изменению тока в катушке (обмотке двигателя в данном случае).

В полупроводниковых ключах дело обстоит более критично – даже небольшая индуктивность в цепи может привести к их пробою, при достижении пиковых значений Uкэ или Uси. Для их защиты устанавливают снабберные цепи, на которых и рассеивается энергия индуктивных всплесков.

Заключение

Подведем итоги. Условиями возникновения ЭДС самоиндукции является: наличие индуктивности в цепи и изменение тока в нагрузке. Это может происходить как в работе, при смене режимов или возмущающих воздействиях, так и при коммутации приборов. Это явление может нанести вред контактам реле и пускателей, так как приводит к при размыкании индуктивных цепей, например, электродвигателей. Чтобы снизить негативное влияние большая часть коммутационной аппаратуры оснащается дугогасительными камерами.

В полезных целях явление ЭДС используется довольно часто, от фильтра для сглаживания пульсаций тока и фильтра частот в аудиоаппаратуре, до трансформаторов и высоковольтных катушек зажигания в автомобилях.

Надеемся, теперь вам стало понятно, что такое самоиндукция, как она проявляется и где ее можно использовать. Если возникли вопросы, задавайте их в комментариях под статьей!

Материалы

Электромагнитная индукция

Магнитный поток. В однородном магнитном поле, модуль вектора индукции которого равен В, помещен плоский замкнутый контур площадью S. Нормаль n к плоскости контура составляет угол a с направлением вектора магнитной индукции В (см. рис. 1).
Магнитным потоком через поверхность называется величина Ф, определяемая соотношением:
Ф = В·S·cos a.         
      
Единица измерения магнитного потока в систем СИ – 1 Вебер (1 Вб).

Электромагнитная индукция. Явление электромагнитной индукции обнаружено в 1831 г. Фарадеем. Оно выражает взаимосвязь электрических и магнитных явлений.
Рассмотрим некоторые экспериментальные факты:

постоянный магнит вставляют в катушку, замкнутую на гальванометр, или вынимают из нее. При движении магнита в контуре возникает электрический ток

Аналогичный результат будет иметь место в случае перемещения электромагнита, по которому пропускают постоянный ток, относительно первичной катушки или при изменении тока в неподвижной вторичной катушке.


рамку, замкнутую на гальванометр, помещают в однородное магнитное поле и вращают. В рамке возникает электрический ток. Если же рамка движется поступательно, не пересекая силовых линий, то ток в ней не возникает.

рамка движется  в неоднородном магнитном поле. Число линий индукции, пересекающих рамку, изменяется. В рамке возникает электрический ток

Ток, возникающий в контуре при изменении магнитного потока, называют индукционным током.
Вы знаете, что условием существования электрического тока в замкнутом контуре является наличие электродвижущей силы, поддерживающей разность потенциалов. Следовательно, при изменении магнитного потока, пронизывающего замкнутый контур, в нем возникает ЭДС, которую называют ЭДС индукции (ei).

Явление возникновения ЭДС в контуре при изменении магнитного потока, пронизывающего контур, называется электромагнитной индукцией.
Если контур замкнут, то ЭДС индукции проявляется в возникновении электрического индукционного тока
I = ei/R , где R- сопротивление контура.
Если контур разомкнут, то на концах проводника возникает разность потенциалов, равная ei.
Направление индукционного тока в контуре определяется правилом Ленца:
Индукционный ток направлен так, чтобы своим магнитным полем противодействовать изменению магнитного потока, которым он вызван.
Направление индукционного тока определяется следующим образом:

   1. установить направление внешнего магнитного поля В.
   2. определить увеличивается или уменьшается поток вектора магнитной индукции внешнего поля.
   3. по правилу Ленца указать направление вектора магнитной индукции индукционного тока Вi.
   4. по правилу правого винта определить направление индукционного тока в контуре.

ЭДС индукции в движущемся проводнике. Пусть проводник длиной L перемещается со скоростью V в однородном магнитном поле, пересекая силовые линии. Вместе с проводником движутся заряды, находящиеся в проводнике. На движущийся в магнитном поле заряд действует сила Лоренца. Свободные электроны смещаются к одному концу проводника, а на другом остаются нескомпенсированные положительные заряды. Возникает разность потенциалов, которая и представляет собой ЭДС индукции ei. Ее величину можно определить, рассчитав работу, совершаемую силой Лоренца при перемещении заряда вдоль проводника:
ei = A/q = F·L/q.

Отсюда следует, что
ei = B·V·L·sin a.

Самоиндукция является частным случаем разнообразных проявлений электромагнитной индукции.

Рассмотрим контур, подключенный к источнику тока. По контуру протекает электрический  ток I. Этот ток создает в окружающем пространстве магнитное поле. В результате контур пронизывается собственным магнитным потоком Ф. Очевидно, что собственный магнитный поток пропорционален току в контуре, создавшему магнитной поле:
Ф = L·I.

Коэффициент пропорциональности L называется индуктивностью контура. Индуктивность зависит от размеров, формы проводника, магнитных свойств среды. Единица измерения индуктивности в системе СИ – 1 Генри (Гн).
Если ток в контуре изменяется, то изменяется и собственный магнитный поток Фс. Изменение величины Фс приводит к возникновению в контуре ЭДС индукции. Данное явление называется самоиндукцией, а соответствующее значение – ЭДС самоиндукции eiс.
Из закона электромагнитной индукции следует, что
eiс = dФс/dt.

Если L = const, то eiс= – L·dI/dt.

Явление самоиндукции формула. Формула эдс индукции

При изменении тока в контуре меняется поток магнитной индукции через поверхность , ограниченную этим контуром, изменение потока магнитной индукции приводит к возбуждению ЭДС самоиндукции. Направление ЭДС оказывается таким, что при увеличении тока в цепи эдс препятствует возрастанию тока, а при уменьшении тока – убыванию.

Величина ЭДС пропорциональна скорости изменения силы тока I и индуктивности контура L :

.

За счёт явления самоиндукции в электрической цепи с источником ЭДС при замыкании цепи ток устанавливается не мгновенно, а через какое-то время. Аналогичные процессы происходят и при размыкании цепи , при этом величина ЭДС самоиндукции может значительно превышать ЭДС источника. Чаще всего в обычной жизни это используется в катушках зажигания автомобилей. Типичное напряжение самоиндукции при напряжении питающей батареи 12В составляет 7-25кВ.

Wikimedia Foundation . 2010 .

Смотреть что такое “ЭДС самоиндукции” в других словарях:

    эдс самоиндукции – — [Я.Н.Лугинский, М.С.Фези Жилинская, Ю.С.Кабиров. Англо русский словарь по электротехнике и электроэнергетике, Москва, 1999 г.] Тематики электротехника, основные понятия EN self induced emfFaraday voltageinductance voltageself induction… …

    Это явление возникновения ЭДС индукции в проводящем контуре при изменении протекающего через контур тока. При изменении тока в контуре пропорционально меняется и магнитный поток через поверхность, ограниченную этим контуром. Изменение… … Википедия

    – (от лат. inductio наведение, побуждение), величина, характеризующая магн. св ва электрич. цепи. Ток, текущий в проводящем контуре, создаёт в окружающем пр ве магн. поле, причём магнитный поток Ф, пронизывающий контур (сцепленный с ним), прямо… … Физическая энциклопедия

    реактивная мощность – Величина, равная при синусоидальных электрическом токе и электрическом напряжении произведению действующего значения напряжения на действующее значение тока и на синус сдвига фаз между напряжением и током двухполюсника. [ГОСТ Р 52002 2003]… … Справочник технического переводчика

    Раздел физики, охватывающий знания о статическом электричестве, электрических токах и магнитных явлениях. ЭЛЕКТРОСТАТИКА В электростатике рассматриваются явления, связанные с покоящимися электрическими зарядами. Наличие сил, действующих между… … Энциклопедия Кольера

    Электрический машина, не имеющая подвижных частей и преобразующая переменный ток одного напряжения в переменный ток другого напряжения. В простейшем случае состоит из магнитопровода (сердечника) и расположенных на нём двух обмоток первичной и… … Энциклопедический словарь

Взаимосвязь электрических и магнитных полей

Электрические и магнитные явления изучались давно, вот только никому не приходило в голову каким-то образом связать эти исследования между собой. И только в 1820 году было обнаружено, что проводник с током действует на стрелку компаса. Это открытие принадлежало датскому физику Хансу Кристиану Эрстеду. Впоследствии его именем была названа единица измерения напряженности магнитного поля в системе СГС: русское обозначение Э (Эрстед), англоязычное – Oe. Такую напряженность магнитное поле имеет в вакууме при индукции в 1 Гаусс.

Это открытие наводило на мысль о том, что из электрического тока можно получить магнитное поле. Но вместе с тем возникали мысли и по поводу обратного преобразования, а именно, как из магнитного поля получить электрический ток. Ведь многие процессы в природе обратимы: из воды получается лед, который можно снова растопить в воду.

На изучение этого очевидного сейчас закона физики после открытия Эрстеда ушло целых двадцать два года. Получением электричества из магнитного поля занимался английский ученый Майкл Фарадей. Делались различной формы и размеров проводники и магниты, искались варианты их взаимного расположения. И только, видимо, случайно ученый обнаружил, что для получения на концах проводника ЭДС необходимо еще одно слагаемое – движение магнита, т.е. магнитное поле должно быть обязательно переменным.

Сейчас это никого уже не удивляет. Именно так работают все электрические генераторы, – пока его чем-то вращают, электроэнергия вырабатывается, лампочка светит. Остановили, перестали вращать, и лампочка погасла.

Электромагнитная индукция

Таким образом, ЭДС на концах проводника возникает лишь в том случае, если его определенным образом перемещать в магнитном поле. Или, точнее говоря, магнитное поле обязательно должно изменяться, быть переменным. Это явление получило название электромагнитной индукции, по-русски электромагнитное наведение: в этом случае говорят, что в проводнике наводится ЭДС. Если к такому источнику ЭДС подключить нагрузку, то в цепи будет протекать ток.

Величина наведенной ЭДС зависит от нескольких факторов: длины проводника, индукции магнитного поля B, и в немалой степени от скорости перемещения проводника в магнитном поле. Чем быстрее вращать ротор генератора, тем напряжение на его выходе выше.

Замечание: электромагнитную индукцию (явление возникновение ЭДС на концах проводника в переменном магнитном поле) не следует путать с магнитной индукцией – векторной физической величиной характеризующей собственно магнитное поле.

Индукция

Этот способ был рассмотрен . Достаточно перемещать проводник в магнитном поле постоянного магнита, или наоборот перемещать (практически всегда вращением) магнит около проводника. Оба варианта однозначно позволят получить переменное магнитное поле. В этом случае способ получения ЭДС называется индукцией. Именно индукция используется для получения ЭДС в различных генераторах. В опытах Фарадея в 1831 году магнит поступательно перемещался внутри катушки провода.

Взаимоиндукция

Это название говорит о том, что в этом явлении принимают участие два проводника. В одном из них протекает изменяющийся ток, который создает вокруг него переменное магнитное поле. Если рядом находится еще один проводник, то на его концах возникает переменная же ЭДС.

Такой способ получения ЭДС называется взаимоиндукцией. Именно по принципу взаимоиндукции работают все трансформаторы, только проводники у них выполнены в виде катушек, а для усиления магнитной индукции применяются сердечники из ферромагнитных материалов.

Если ток в первом проводнике прекратится (обрыв цепи), или станет пусть даже очень сильным, но постоянным (нет никаких изменений), то на концах второго проводника никакой ЭДС получить не удастся. Вот почему трансформаторы работают только на переменном токе: если к первичной обмотке подключить гальваническую батарейку, то на выходе вторичной обмотки никакого напряжения однозначно не будет.

ЭДС во вторичной обмотке наводится только при изменении магнитного поля. Причем, чем сильнее скорость изменения, именно скорость, а не абсолютная величина, тем больше будет наведенная ЭДС.

Самоиндукция

Если убрать второй проводник, то магнитное поле в первом проводнике будет пронизывать не только окружающее пространство, но и сам проводник. Таким образом, под воздействием своего поля в проводнике наводится ЭДС, которая называется ЭДС самоиндукции.

Явления самоиндукции в 1833 году изучал русский ученый Ленц. На основании этих опытов удалось выяснить интересную закономерность: ЭДС самоиндукции всегда противодействует, компенсирует внешнее переменное магнитное поле, которое вызывает эту ЭДС. Эта зависимость называется правилом Ленца (не путать с законом Джоуля – Ленца).

Знак «минус» в формуле как раз и говорит о противодействии ЭДС самоиндукции причинам ее породившим. Если катушку подключить к источнику постоянного тока, ток будет возрастать достаточно медленно. Это очень заметно при «прозвонке» первичной обмотки трансформатора стрелочным омметром: скорость движения стрелки в сторону нулевого деления шкалы заметно меньше, чем при проверке резисторов.

При отключении катушки от источника тока ЭДС самоиндукции вызывает искрение контактов реле. В случае, когда катушка управляется транзистором, например катушка реле, то параллельно ей ставится диод в обратном направлении по отношению к источнику питания. Это делается для того, чтобы защитить полупроводниковые элементы от воздействия ЭДС самоиндукции, которая может в десятки и даже сотни раз превышать напряжение источника питания.

Для проведения опытов Ленц сконструировал интересный прибор. На концах алюминиевого коромысла закреплены два алюминиевых же кольца. Одно кольцо сплошное, а в другом был сделан пропил. Коромысло свободно вращалось на иголке.

При введении постоянного магнита в сплошное кольцо оно «убегало» от магнита, а при выведении магнита стремилось за ним. Те же самые действия с разрезанным кольцом никаких движений не вызывали. Это объясняется тем, что в сплошном кольце под воздействием переменного магнитного поля возникает ток, который создает магнитное поле. А в разомкнутом кольце тока нет, следовательно, нет и магнитного поля.

Немаловажная деталь этого опыта в том, что если магнит будет введен в кольцо и останется неподвижным, то никакой реакции алюминиевого кольца на присутствие магнита не наблюдается. Это лишний раз подтверждает, что ЭДС индукции возникает только в случае изменения магнитного поля, причем величина ЭДС зависит от скорости изменения. В данном случае просто от скорости перемещения магнита.

То же можно сказать и о взаимоиндукции и самоиндукции, только изменение напряженности магнитного поля, точнее скорость его изменения зависит от скорости изменения тока. Для иллюстрации этого явления можно привести такой пример.

Пусть через две достаточно большие одинаковые катушки проходят большие токи: через первую катушку 10А, а через вторую целых 1000, причем в обеих катушках токи линейно возрастают. Предположим, что за одну секунду ток в первой катушке изменился с 10 до 15А, а во второй с 1000 до 1001А, что вызвало появление ЭДС самоиндукции в обеих катушках.

Но, несмотря на такое огромное значение тока во второй катушке, ЭДС самоиндукции будет больше в первой, поскольку там скорость изменения тока 5А/сек, а во второй всего 1А/сек. Ведь ЭДС самоиндукции зависит от скорости возрастания тока (читай магнитного поля), а не от его абсолютной величины.

Индуктивность

Магнитные свойства катушки с током зависят от количества витков, геометрических размеров. Значительного усиления магнитного поля можно добиться введением в катушку ферромагнитного сердечника. О магнитных свойствах катушки с достаточной точностью можно судить по величине ЭДС индукции, взаимоиндукции или самоиндукции. Все эти явления были рассмотрены выше.

Характеристика катушки, которая рассказывает об этом, называется коэффициентом индуктивности (самоиндукции) или просто индуктивностью. В формулах индуктивность обозначается буквой L, а на схемах этой же буквой обозначаются катушки индуктивности.

Единица измерения индуктивности – генри (Гн). Индуктивностью 1Гн обладает катушка, в которой при изменении тока на 1А в секунду вырабатывается ЭДС 1В. Это величина достаточно большая: индуктивностью в один и более Гн обладают сетевые обмотки достаточно мощных трансформаторов.

Поэтому достаточно часто пользуются величинами меньшего порядка, а именно милли и микро генри (мГн и мкГн). Такие катушки применяются в электронных схемах. Одно из применений катушек – колебательные контура в радиоустройствах.

Также катушки используются в качестве дросселей, основное назначение которых пропустить без потерь постоянный ток при этом ослабив переменный (фильтры ). Как правило, чем выше рабочая частота, тем меньшей индуктивности требуются катушки.

Индуктивное сопротивление

Если взять достаточно мощный сетевой трансформатор и сопротивление первичной обмотки, то окажется, что оно всего несколько Ом, и даже близко к нулю. Выходит, что ток через такую обмотку будет очень большим, и даже стремиться к бесконечности. Кажется, короткое замыкание просто неизбежно! Так почему же его нет?

Одним из основных свойств катушек индуктивности является индуктивное сопротивление, которое зависит от индуктивности и от частоты переменного тока, который подведен к катушке.

Нетрудно видеть, что с увеличением частоты и индуктивности индуктивное сопротивление увеличивается, а на постоянном токе вообще становится равным нулю. Поэтому при измерении сопротивления катушек мультиметром измеряется только активное сопротивление провода.

Конструкция катушек индуктивности весьма разнообразна и зависит от частот, на которых работает катушка. Например, для работы в дециметровом диапазоне радиоволн достаточно часто используются катушки, выполненные печатным монтажом. При массовом производстве такой способ очень удобен.

Индуктивность катушки зависит от ее геометрических размеров, сердечника, количества слоев и формы. В настоящее время выпускается достаточное количество стандартных катушек индуктивности похожих на обычные резисторы с выводами. Маркировка таких катушек выполняется цветными кольцами. Также существуют катушки для поверхностного монтажа, применяемые в качестве дросселей. Индуктивность таких катушек составляет несколько миллигенри.

Термин индукция в электротехнике означает возникновение тока в электрической замкнутой цепи, если она находится в изменяющемся Открыта всего-то двести лет назад Майклом Фарадеем. Значительно раньше это мог бы сделать Андре Ампер, проводивший похожие опыты. Он вставлял в катушку металлический стержень, а затем, вот незадача, шел в другую комнату посмотреть на стрелку гальванометра – а вдруг она шевельнется. А стрелка исправно делала свое дело – отклонялась, но пока Ампер странствовал по комнатам – возвращалась на нулевую отметку. Вот так явление самоиндукции дожидалось еще добрый десяток лет, пока катушка, прибор и исследователь окажутся одновременно в нужном месте.

Главным моментом этого эксперимента было то, что ЭДС индукции возникает только тогда, когда магнитное поле, проходящее через замкнутый контур, изменяется. А вот менять его можно как угодно – или изменять величину самого магнитного поля, или просто перемещать источник поля относительно того же замкнутого контура. ЭДС, которая при этом возникает, назвали “ЭДС взаимоиндукции”. Но это было только начало открытий в области индукции. Еще более удивительным было явление самоиндукции, которое открыл примерно в то же время. В его опытах было обнаружено, что катушки не только индуцировало ток в другой катушке, но и при изменении тока в этой катушке, наводило в ней же дополнительную ЭДС. Вот ее-то и назвали ЭДС самоиндукции. В большое интерес представляет направление тока. Оказалось, что в случае с ЭДС самоиндукции ее ток направлен против своего “родителя” – тока, обусловленного основной ЭДС.

А можно наблюдать явление самоиндукции? Как говорится, нет ничего проще. Соберем две первая – последовательно включенная катушка индуктивности и лампочка, а вторая – только лампочка. Подключим их к аккумулятору через общий выключатель. При включении можно видеть, что лампочка в цепи с катушкой загорается “нехотя”, а вторая лампочка, более быстрая “на подъем”, включается мгновенно. Что происходит? В обеих цепях после включения начинает протекать ток, причем он изменяется от нуля до своего максимума, а как раз изменения тока и дожидается катушка индуктивности, которая порождает ЭДС самоиндукции. Есть ЭДС и замкнутая цепь – значит, есть и ее ток, но направлен он противоположно основному току цепи, который, в конце концов, достигнет максимального значения, определяемого параметрами цепи, и перестанет расти, а раз нет изменения тока – нет и ЭДС самоиндукции. Все просто. Аналогичная картина, но с “точностью до наоборот”, наблюдается при выключении тока. Верная своей “вредной привычке” противодействовать любому изменению тока, ЭДС самоиндукции поддерживает его протекание в цепи после отключения питания.

Сразу же стал вопрос – в чем заключается явление самоиндукции? Было установлено, что на ЭДС самоиндукции влияет скорость изменения тока в проводнике, и можно записать:

Отсюда видно, что ЭДС самоиндукции Е прямопропорциональна скорости изменения тока dI/dt и коэффициенту пропорциональности L, названному индуктивностью. За свой вклад в исследование вопроса, в чем состоит явление самоиндукции, Джордж Генри был вознагражден тем, что его имя носит единица измерения индуктивности — генри (Гн). Именно индуктивность цепи протекания тока определяет явление самоиндукции. Можно представить, что индуктивность – это некое “хранилище” магнитной энергии. В случае увеличения тока в цепи электрическая энергия преобразуется в магнитную, задерживает рост тока, а при уменьшении тока магнитная энергия катушки преобразуется в электрическую и поддерживает ток в цепи.

Наверное, каждому приходилось видеть искру при выключении вилки из розетки – это самый распространенный вариант проявления ЭДС самоиндукции в реальной жизни. Но в быту размыкаются токи максимум 10-20 А, а время размыкания порядка 20 мсек. При индуктивности порядка 1 Гн ЭДС самоиндукции в этом случае будет равна 500 В. Казалось бы, что вопрос, в чем состоит явление самоиндукции, не так и сложен. А на самом деле, ЭДС самоиндукции представляет собой большую техническую проблему. Суть в том, что при разрыве цепи, когда контакты уже разошлись, самоиндукция поддерживает протекание тока, а это приводит к выгоранию контактов, т.к. в технике коммутируются цепи с токами в сотни и даже тысячи ампер. Здесь зачастую речь идет об ЭДС самоиндукции в десятки тысяч вольт, а это требует дополнительного решения технических вопросов, связанных с перенапряжениями в электрических цепях.

Но не все так мрачно. Бывает, что эта вредная ЭДС очень даже полезна, например, в системах зажигания ДВС. Такая система состоит из катушки индуктивности в виде автотрансформатора и прерывателя. Через первичную обмотку пропускается ток, который выключается прерывателем. В результате обрыва цепи возникает ЭДС самоиндукции в сотни вольт (при этом аккумулятор дает всего 12В). Дальше это напряжение дополнительно трансформируется, и на свечи зажигания поступает импульс больше 10 кВ.

Знаете ли Вы, в чем ложность понятия “физический вакуум”?

Физический вакуум – понятие релятивистской квантовой физики, под ним там понимают низшее (основное) энергетическое состояние квантованного поля, обладающее нулевыми импульсом, моментом импульса и другими квантовыми числами. Физическим вакуумом релятивистские теоретики называют полностью лишённое вещества пространство, заполненное неизмеряемым, а значит, лишь воображаемым полем. Такое состояние по мнению релятивистов не является абсолютной пустотой, но пространством, заполненным некими фантомными (виртуальными) частицами. Релятивистская квантовая теория поля утверждает, что, в согласии с принципом неопределённости Гейзенберга, в физическом вакууме постоянно рождаются и исчезают виртуальные, то есть кажущиеся (кому кажущиеся?), частицы: происходят так называемые нулевые колебания полей. Виртуальные частицы физического вакуума, а следовательно, он сам, по определению не имеют системы отсчета, так как в противном случае нарушался бы принцип относительности Эйнштейна, на котором основывается теория относительности (то есть стала бы возможной абсолютная система измерения с отсчетом от частиц физического вакуума, что в свою очередь однозначно опровергло бы принцип относительности, на котором постороена СТО). Таким образом, физический вакуум и его частицы не есть элементы физического мира, но лишь элементы теории относительности, которые существуют не в реальном мире, но лишь в релятивистских формулах, нарушая при этом принцип причинности (возникают и исчезают беспричинно), принцип объективности (виртуальные частицы можно считать в зависимсоти от желания теоретика либо существующими, либо не существующими), принцип фактической измеримости (не наблюдаемы, не имеют своей ИСО).

Когда тот или иной физик использует понятие “физический вакуум”, он либо не понимает абсурдности этого термина, либо лукавит, являясь скрытым или явным приверженцем релятивистской идеологии.

Понять абсурдность этого понятия легче всего обратившись к истокам его возникновения. Рождено оно было Полем Дираком в 1930-х, когда стало ясно, что отрицание эфира в чистом виде, как это делал великий математик, но посредственный физик , уже нельзя. Слишком много фактов противоречит этому.

Для защиты релятивизма Поль Дирак ввел афизическое и алогичное понятие отрицательной энергии, а затем и существование “моря” двух компенсирующих друг друга энергий в вакууме – положительной и отрицательной, а также “моря” компенсирующих друг друга частиц – виртуальных (то есть кажущихся) электронов и позитронов в вакууме.

Магнитное поле контура, в котором сила тока изменяется, индуцирует ток не только в других контурах, но и в себе самом. Это явление получило название самоиндукции.

Опытным путём установлено, что магнитный поток вектора магнитной индукции поля, создаваемого текущим в контуре током, пропорционален силе этого тока:

где L– индуктивность контура. Постоянная характеристика контура, которая зависит от его формы и размеров, а так же от магнитной проницаемости среды, в которой находится контур. [L] = Гн (Генри,

1Гн = Вб/А).

Если за время dtток в контуре изменится наdI, то магнитный поток, связанный с этим током, изменится наdФ =LdIв результате чего в этом контуре появится ЭДС самоиндукции:

Знак минус показывает, что ЭДС самоиндукции (а, следовательно, и ток самоиндукции) всегда препятствует изменению силы тока, который вызвал самоиндукцию.

Наглядным примером явления самоиндукции служат экстратоки замыкания и размыкания, возникающие при включении и выключении электрических цепей, обладающей значительной индуктивностью.

Энергия магнитного поля

Магнитное поле обладает потенциальной энергией, которая в момент его образования (или изменения) пополняется за счёт энергии тока в цепи, совершающего при этом работу против ЭДС самоиндукции, возникающей вследствие изменения поля.

Работа dAза бесконечно малый промежуток времениdt, в течении которого ЭДС самоиндукциии токIможно считать постоянными, равняется:

. (5)

Знак минус указывает, что элементарная работа совершается током против ЭДС самоиндукции. Чтобы определить работу при изменении тока от 0 до I, проинтегрируем правую часть, получим:

. (6)

Эта работа численно равна приросту потенциальной энергии ΔW п магнитного поля, связанного с этой цепью, т.е.A= -ΔW п.

Выразим энергию магнитного поля через его характеристики на примере соленоида. Будем считать, что магнитное поле соленоида однородно и в основном расположено внутри его. Подставим в (5) значение индуктивности соленоида, выраженное через его параметры и значение силы тока I, выраженное из формулы индукции магнитного поля соленоида:

, (7)

где N – общее число витков соленоида; ℓ – его длина; S – площадь сечения внутреннего канала соленоида.

, (8)

После подстановки имеем:

Разделив обе части на V, получим объёмную плотность энергии поля:

(10)

или, с учётом, что
получим,
. (11)

Переменный ток

2.1 Переменный ток и его основные характеристики

Переменным называется ток, изменяющийся с течением времени и по величине и по направлению. Примером переменного тока может служить потребляемый промышленный ток. Этот ток является синусоидальным, т.е. мгновенное значение его параметров меняются со временем по закону синуса (или косинуса):

i = I 0 sinωt, u = U 0 sin(ωt + φ 0). (12)

Переменный синусоидальный ток можно получить, если вращать рамку (контур) с постоянной скоростью

в однородном магнитном поле с индукцией B (рис.5). При этом магнитный поток, пронизывающий контур, изменяется по закону

где S– площадь контура, α = ωt– угол поворота рамки за время t. Изменение потока приводит к возникновению ЭДС индукции

, (17)

направление которой определяется по правилу Ленца.

Если контур замкнут (рис.5), то по нему идёт ток:

. (18)

График изменения электродвижущей силыи индукционного токаi представлен на рис.6.

Переменный ток характеризуется периодом Т, частотой ν = 1/Т, циклической частотой
и фазой φ = (ωt + φ 0) Графически значения напряжения и силы переменного тока на участке цепи будут представляться двумя синусоидами, в общем случае сдвинутыми по фазе на φ.

Для характеристики переменного тока вводятся понятия действующего (эффективного) значения тока и напряжения. Эффективным значением силы переменного тока называется сила такого постоянного тока, который выделяет в данном проводнике столько же тепла за время одного периода, сколько выделяет тепла и данный переменный ток.

,
. (13)

Приборы, включенные в цепь переменного тока (амперметр, вольтметр), показывают эффективные значения тока и напряжения.

Собственная индуктивность

Собственная индуктивность
Далее: Энергия, накопленная в Up: индуктивность Предыдущее: Взаимная индуктивность


Самостоятельная индуктивность Нам не обязательно нужны две цепи для индуктивного эффекта. Рассмотреть возможность одиночная проводящая цепь, вокруг которой течет ток течет. Этот ток создает магнитное поле, которое вызывает магнитный поток, связывающий схема. Мы ожидаем, что поток будет прямо пропорционален току, учитывая линейный характер законов магнитостатики, и определение магнитного потока.Таким образом, мы можем написать
(241)

где константа пропорциональности называется самоиндукцией схема. Как и взаимная индуктивность, самоиндукция цепи измеряется в единицах генри, и является чисто геометрическая величина, зависящая только от форма цепи и количество витков в цепи.

Если ток, протекающий по цепи, изменяется на количество во временном интервале, то магнитный поток, соединяющий цепь, изменяется на величину в том же временном интервале.В соответствии с Закон Фарадея, ЭДС

(242)

генерируется по контуру. С , эту ЭДС также можно записать
(243)

Таким образом, ЭДС, генерируемая вокруг цепи за счет собственного тока, напрямую пропорционально скорости изменения тока. Закон Ленца и здравый смысл, требуйте, чтобы, если ток увеличивается, то ЭДС должна всегда действовать для уменьшения тока, а наоборот .Это легко оценить, так как если ЭДС действовала на увеличение ток, когда ток возрастал, мы явно получили бы нефизический положительный отзыв эффект, при котором ток продолжал неограниченно расти. Следовательно, из Уравнение (243), что собственная индуктивность цепи обязательно равна положительному числу . Этот это не относится к взаимной индуктивности, которая может быть как положительной, так и отрицательной.

Рассмотрим соленоид длины и сечения площадь .Предположим, что у соленоида есть витки. Когда в соленоиде течет ток, возникает однородное осевое поле величиной

(244)

генерируется в сердечнике соленоида. Напряженность поля вне ядра является незначительный. Магнитный поток, связывающий один виток соленоида, равен . Таким образом, магнитный поток, связывающий все витки соленоид
(245)

Согласно формуле.(241) собственная индуктивность соленоида определяется выражением , что сводится к
(246)

Обратите внимание, что это положительно. Кроме того, это геометрическая величина, зависящая от только от размеров соленоида, и количества витков в соленоиде.

Инженеры хотели бы уменьшить все части электрического оборудования, независимо от того, насколько они сложны, до эквивалентная схема , состоящая из сети всего четыре разных типов компонента.Этими четырьмя основными компонентами являются ЭДС , резисторы , конденсаторы , и индукторы . Катушка индуктивности – это просто чистая самоиндукция, и обычно она На принципиальных схемах представлен небольшой соленоид. На практике индукторы обычно состоят из коротких соленоидов с воздушным сердечником, намотанных из эмалированной медной проволоки.



Далее: Энергия, накопленная в Up: индуктивность Предыдущее: Взаимная индуктивность
Ричард Фицпатрик 2007-07-14

Собственная индуктивность

Самоиндукция
Далее: Взаимная индуктивность Up: Магнитная индукция Предыдущее: Индуктивность Рассмотрим длинный соленоид длины и радиуса, который имеет количество витков на единицу длины, и несет ток.Продольный (, т. Е. , направленный по ось соленоида) магнитное поле внутри соленоида примерно однородное, и дается
(907)

Этот результат легко получить, интегрировав закон Ампера по прямоугольной петля, длинные стороны которой проходят параллельно оси соленоида, одна внутри соленоид, а другой снаружи, короткие стороны которого перпендикулярны ось. Магнитный поток через каждый виток контура равен .Общий поток через провод соленоида с витками
(908)

Таким образом, самоиндукция соленоида равна
(909)

Обратите внимание, что самоиндукция зависит только от геометрических величин, таких как число витков на единицу длины соленоида и площадь поперечного сечения витков.

Предположим, что ток, протекающий через соленоид, изменился.Мы должны Предположим, что изменение достаточно медленное, поэтому смещением можно пренебречь. в наших расчетах эффекты тока и запаздывания. Это означает, что типичный масштаб времени изменения должен быть намного больше, чем время, необходимое лучу света, чтобы пройти через схема. Если это так, то приведенные выше формулы остаются в силе.

Изменение тока подразумевает изменение магнитного потока, связывающего соленоид проволока, так как . По мнению Фарадея закон, это изменение генерирует e.м.ф. в проводе. По закону Ленца э.д.с. это так что касается противодействия смене нынешней – т.е., то это обратная ЭДС. Мы можем написать

(910)

где – сгенерированная ЭДС.
Рисунок 51:
Предположим, что у нашего соленоида есть электрическое сопротивление. Позволь нам подключите концы соленоида к клеммам аккумуляторной батареи. е.м.ф. . Что сейчас произойдет? Эквивалентная схема показана на рис.51. Индуктивность и сопротивление соленоида представлены идеальным индуктор“ и идеальный резистор“, соединенные последовательно. Падение напряжения на катушке индуктивности и резисторе равна ЭДС. батареи, . Падение напряжения на резисторе просто, тогда как падение напряжения на катушке индуктивности (, т. е. , обратная ЭДС) составляет . Здесь – ток, протекающий через соленоид. Следует, что
(911)

Это дифференциальное уравнение для тока.Мы можем изменить это на давать
(912)

Общее решение
(913)

Константа фиксируется граничными условиями. Предположим, что аккумулятор подключается в то время, когда. Отсюда следует, что, поэтому что
(914)

Эта кривая изображена на рис.52. Видно, что после подключения АКБ ток возрастает и достигает своего установившегося значения (которое исходит от закон), на характерном масштабе времени
(915)

Эта шкала времени иногда называется постоянной времени схемы, или несколько невообразимо, L по времени R схемы.
Рисунок 52:

Теперь мы можем оценить значение самоиндукции.Задний Э.д.с. генерируется в катушке индуктивности, поскольку ток пытается измениться, эффективно предотвращает ток от нарастания (или падения) намного быстрее, чем время. Этот эффект иногда выгодно, но часто это очень неприятно. Все элементы схемы обладают некоторой самоиндукцией, а также некоторым сопротивлением и, следовательно, имеют конечное время. Это означает, что когда мы включаем цепь, ток не подскакивает мгновенно до своего установившегося значения. Вместо этого нарастание распределяется по времени L / R цепи.Это хорошая вещь. Если бы ток увеличивался мгновенно, тогда чрезвычайно большое электрическое поля будут генерироваться внезапным скачком индуцированного магнитного поля, ведущего, неизбежно к пробою и возникновению электрической дуги. Итак, если бы такого не было как самоиндукция, то каждый раз, когда вы включаете или выключаете электрическую цепь будет синяя вспышка из-за дуги между проводниками. Самоиндуктивность тоже может быть плохо. Предположим, у нас есть необычный блок питания, и мы хотим использовать его для передачи электрического сигнала по проводу (или линии передачи).Конечно, провод или линия передачи будут обладать как сопротивлением, так и индуктивностью, и, следовательно, будет иметь некоторое характерное время. Предположим, что мы попробуйте послать прямоугольный сигнал по проводу. Поскольку ток в проводе не может подниматься или опускаться быстрее времени, передний и задний края сигнал со временем сглаживается. Типичная разница между сигнал, поступающий в провод (верхний график), и тот, который выходит из другой конец (нижняя кривая) изображен на рис.53. Ясно, что мало Дело в том, что у вас есть необычный источник питания, если только вы не обладаете низкой индуктивностью провод или линия передачи, так что сигнал от источника питания может быть передается на какое-то нагрузочное устройство без серьезных искажений.

Рисунок 53:


Далее: Взаимная индуктивность Up: Магнитная индукция Предыдущее: Индуктивность
Ричард Фицпатрик 2006-02-02

23.9 Индуктивность – физика колледжа: OpenStax

Индукция – это процесс, при котором ЭДС индуцируется изменением магнитного потока. До сих пор обсуждалось множество примеров, некоторые из которых более эффективны, чем другие. Трансформаторы, например, спроектированы так, чтобы быть особенно эффективными при наведении желаемого напряжения и тока с очень небольшими потерями энергии в другие формы. Есть ли полезная физическая величина, связанная с тем, насколько «эффективно» данное устройство? Ответ положительный, и эта физическая величина называется , индуктивность .

Взаимная индуктивность – это влияние закона индукции Фарадея для одного устройства на другое, например, первичная катушка, при передаче энергии вторичной обмотке в трансформаторе. См. Рис. 1, где простые катушки индуцируют ЭДС друг в друге.

Рисунок 1. Эти катушки могут наводить ЭДС друг в друге, как неэффективный трансформатор. Их взаимная индуктивность M указывает на эффективность связи между ними. Здесь видно, что изменение тока в катушке 1 вызывает ЭДС в катушке 2.(Обратите внимание, что « E 2 индуцированная» представляет наведенную ЭДС в катушке 2.)

Во многих случаях, когда геометрия устройств является фиксированной, магнитный поток изменяется путем изменения тока. Поэтому мы концентрируемся на скорости изменения тока, [латекс] \ boldsymbol {\ Delta I / \ Delta t} [/ latex], как на причине индукции. Изменение текущего [латекса] \ boldsymbol {I_1} [/ latex] в одном устройстве, катушка 1 на рисунке, вызывает [латекс] \ boldsymbol {\ textbf {emf} _2} [/ latex] в другом.Мы выражаем это в форме уравнения как

[латекс] \ boldsymbol {\ textbf {emf} _2 = -M} [/ latex] [латекс] \ boldsymbol {\ frac {\ Delta I_1} {\ Delta t}} [/ latex],

где [латекс] \ boldsymbol {M} [/ latex] определяется как взаимная индуктивность между двумя устройствами. Знак минус является выражением закона Ленца. Чем больше взаимная индуктивность [латекс] \ boldsymbol {M} [/ latex], тем эффективнее связь. Например, катушки на рисунке 1 имеют небольшой [латекс] \ boldsymbol {M} [/ latex] по сравнению с катушками трансформатора в главе 23.7 Рисунок 3. Единицы измерения [латекс] \ boldsymbol {M} [/ latex]: [латекс] \ boldsymbol {(\ textbf {V} \ cdot \; \ textbf {s}) / \ textbf {A} = \ Omega \ cdot \; \ textbf {s}} [/ latex], который назван henry (H) в честь Джозефа Генри. То есть [латекс] \ boldsymbol {1 \; \ textbf {H} = 1 \ Omega \ cdot \; \ textbf {s}} [/ latex].

Природа здесь симметрична. Если мы изменим ток [латекс] \ boldsymbol {I_2} [/ latex] в катушке 2, мы индуцируем [латекс] \ boldsymbol {\ textbf {emf} _1} [/ latex] в катушке 1, что определяется значением

[латекс] \ boldsymbol {\ textbf {emf} _1 = -M} [/ latex] [латекс] \ boldsymbol {\ frac {\ Delta I_2} {\ Delta t}} [/ latex],

где [latex] \ boldsymbol {M} [/ latex] то же самое, что и для обратного процесса.Трансформаторы работают в обратном направлении с такой же эффективностью или взаимной индуктивностью [латекс] \ boldsymbol {M} [/ latex] .

Большая взаимная индуктивность [латекс] \ boldsymbol {M} [/ latex] может быть желательной, а может и нежелательной. Мы хотим, чтобы трансформатор имел большую взаимную индуктивность. Но такой прибор, как электрическая сушилка для одежды, может вызвать опасную ЭДС на корпусе, если взаимная индуктивность между его катушками и корпусом велика. Один из способов уменьшить взаимную индуктивность [латекс] \ boldsymbol {M} [/ latex] – это перемотать катушки, чтобы нейтрализовать создаваемое магнитное поле.(См. Рисунок 2.)

Рис. 2. Нагревательные катушки электрической сушилки для белья могут быть намотаны в противоположную сторону, так что их магнитные поля нейтрализуют друг друга, что значительно снижает взаимную индуктивность по отношению к корпусу сушилки.

Самоиндукция , действие закона индукции Фарадея устройства на самого себя, также существует. Когда, например, увеличивается ток через катушку, магнитное поле и магнитный поток также увеличиваются, вызывая противоэдс, как того требует закон Ленца. И наоборот, если ток уменьшается, индуцируется ЭДС, препятствующая уменьшению.Большинство устройств имеют фиксированную геометрию, поэтому изменение потока происходит полностью из-за изменения тока [latex] \ boldsymbol {\ Delta I} [/ latex] через устройство. Индуцированная ЭДС связана с физической геометрией устройства и скоростью изменения тока. Выдается

[латекс] \ boldsymbol {\ textbf {emf} = -L} [/ latex] [латекс] \ boldsymbol {\ frac {\ Delta I} {\ Delta t}} [/ латекс],

где [латекс] \ boldsymbol {L} [/ latex] – это собственная индуктивность устройства. Устройство, которое демонстрирует значительную самоиндукцию, называется индуктором и обозначено символом на рисунке 3.

Рис. 3.

Знак минус является выражением закона Ленца, означающего, что ЭДС препятствует изменению тока. Единицами самоиндукции является генри (Гн), как и для взаимной индуктивности. Чем больше самоиндукция [латекс] \ boldsymbol {L} [/ latex] устройства, тем сильнее оно сопротивляется любому изменению тока через него. Например, большая катушка с множеством витков и железным сердечником имеет большой [латекс] \ boldsymbol {L} [/ latex] и не позволяет току быстро меняться. Чтобы избежать этого эффекта, необходимо получить небольшой [латекс] \ boldsymbol {L} [/ latex], например, за счет встречной намотки катушек, как на рисунке 2.

Индуктор 1 Гн – это большой индуктор. Чтобы проиллюстрировать это, рассмотрим устройство с [latex] \ boldsymbol {L = 1.0 \; \ textbf {H}} [/ latex], через которое протекает ток 10 A. Что произойдет, если мы попытаемся быстро отключить ток, возможно, всего за 1,0 мс? ЭДС, заданная как [latex] \ boldsymbol {\ textbf {emf} = -L (\ Delta I / \ Delta t)} [/ latex], будет препятствовать изменению. Таким образом, ЭДС будет индуцирована [latex] \ boldsymbol {\ textbf {emf} = -L (\ Delta I / \ Delta t) = (1.0 \; \ textbf {H}) [(10 \; \ textbf { A}) / (1.0 \; \ textbf {ms})] = 10 000 \; \ textbf {V}} [/ latex]. Положительный знак означает, что это большое напряжение направлено в том же направлении, что и ток, но противодействует его уменьшению. Такие большие ЭДС могут вызвать дуги, повредить коммутационное оборудование, и поэтому может потребоваться более медленное изменение тока.

Есть применение для такого большого наведенного напряжения. Во вспышках камеры используются батарея, два индуктора, которые работают как трансформатор, и система переключения или генератор для создания больших напряжений. (Помните, что нам нужно изменяющееся магнитное поле, вызванное изменяющимся током, чтобы вызвать напряжение в другой катушке.) Система генератора будет делать это много раз, когда напряжение батареи повышается до более чем тысячи вольт. (Вы можете услышать пронзительный вой от трансформатора, когда конденсатор заряжается.) Конденсатор сохраняет высокое напряжение для последующего использования для питания вспышки. (См. Рисунок 4.)

Рисунок 4. Благодаря быстрому переключению катушки индуктивности можно использовать батареи 1,5 В для индукции ЭДС в несколько тысяч вольт. Это напряжение можно использовать для хранения заряда в конденсаторе для последующего использования, например, в насадке для вспышки камеры.

Можно рассчитать [латекс] \ boldsymbol {L} [/ latex] для индуктора, учитывая его геометрию (размер и форму) и зная создаваемое магнитное поле. В большинстве случаев это сложно из-за сложности создаваемого поля. Итак, в этом тексте индуктивность [латекс] \ boldsymbol {L} [/ латекс] обычно является заданной величиной. Единственным исключением является соленоид, потому что он имеет очень однородное поле внутри, почти нулевое поле снаружи и простую форму. Поучительно вывести уравнение для его индуктивности.Начнем с того, что наведенная ЭДС определяется законом индукции Фарадея как [latex] \ boldsymbol {\ textbf {emf} = -N (\ Delta \ phi / \ Delta t)} [/ latex] и по определению собственной индуктивности, как [латекс] \ boldsymbol {\ textbf {emf} = – L (\ Delta I / \ Delta t)} [/ latex]. Приравнивая эти доходности к

[латекс] \ boldsymbol {\ textbf {emf} = -N} [/ latex] [латекс] \ boldsymbol {\ frac {\ Delta \ phi} {\ Delta t}} [/ latex] [латекс] \ boldsymbol { = -L} [/ latex] [латекс] \ boldsymbol {\ frac {\ Delta I} {\ Delta t}} [/ latex].

Решение для [latex] \ boldsymbol {L} [/ latex] дает

[латекс] \ boldsymbol {L = N} [/ latex] [латекс] \ boldsymbol {\ frac {\ Delta \ phi} {\ Delta I}} [/ латекс].

Это уравнение для самоиндукции [latex] \ boldsymbol {L} [/ latex] устройства всегда верно. Это означает, что самоиндукция [латекс] \ boldsymbol {L} [/ латекс] зависит от того, насколько эффективен ток для создания магнитного потока; чем эффективнее, тем лучше [латекс] \ boldsymbol {\ Delta \ phi \ Delta I} [/ latex].

Давайте воспользуемся этим последним уравнением, чтобы найти выражение для индуктивности соленоида. Поскольку площадь [латекс] \ boldsymbol {A} [/ latex] соленоида фиксирована, изменение магнитного потока составляет [латекс] \ boldsymbol {\ Delta \ phi = \ Delta (BA) = A \ Delta B} [/ латекс].Чтобы найти [латекс] \ boldsymbol {\ Delta B} [/ latex], отметим, что магнитное поле соленоида задается как [латекс] \ boldsymbol {B = \ mu _0 nI = \ mu 0 \ frac {NI} {\ ell}} [/ латекс]. (Здесь [latex] \ boldsymbol {n = N / \ ell} [/ latex], где [latex] \ boldsymbol {N} [/ latex] – количество витков, а [latex] \ boldsymbol {\ ell} [/ latex] – длина соленоида.) Изменяется только текущий, так что [latex] \ boldsymbol {\ Delta \ phi = A \ Delta B = \ mu_0 NA \ frac {\ Delta I} {\ ell}} [/ latex] . Замена [latex] \ boldsymbol {\ Delta \ phi} [/ latex] на [latex] \ boldsymbol {L = N \ frac {\ Delta \ phi} {\ Delta I}} [/ latex] дает

[латекс] \ boldsymbol {L = N} [/ latex] [латекс] \ boldsymbol {\ frac {\ Delta \ phi} {\ Delta I}} [/ latex] [латекс] \ boldsymbol {= N} [/ латекс] [латекс] \ boldsymbol {\ frac {\ mu_0 NA \ frac {\ Delta I} {\ ell}} {\ Delta I}} [/ латекс].2A} {\ ell}} [/ latex] [латекс] \ boldsymbol {(\ textbf {соленоид})}. [/ Latex]

Это самоиндукция соленоида с площадью поперечного сечения [латекс] \ boldsymbol {A} [/ latex] и длиной [латекс] \ boldsymbol {\ ell} [/ latex]. Обратите внимание, что индуктивность зависит только от физических характеристик соленоида, в соответствии с его определением.

Пример 1: Расчет самоиндукции соленоида среднего размера

Рассчитайте самоиндукцию соленоида длиной 10,0 см и диаметром 4,00 см, который имеет 200 катушек.2)} {0.100 \; \ textbf {m}} \\ [1em] & \ boldsymbol {0.632 \; \ textbf {mH}} \ end {array} [/ latex].

Обсуждение

Этот соленоид среднего размера. Его индуктивность около миллигенри также считается умеренной.

Одно из распространенных применений индуктивности используется в светофорах, которые могут определить, когда автомобили ждут на перекрестке. Электрическая цепь с индуктором размещается на дороге под местом остановки ожидающей машины. Кузов автомобиля увеличивает индуктивность, и схема изменяется, посылая сигнал на светофор, чтобы изменить цвет.Точно так же металлоискатели, используемые для безопасности аэропортов, используют ту же технику. Катушка или индуктор в корпусе металлоискателя действует как передатчик и как приемник. Импульсный сигнал в катушке передатчика вызывает сигнал в приемнике. На самоиндукцию цепи влияет любой металлический предмет на пути. Такие детекторы могут быть настроены на чувствительность, а также могут указывать приблизительное местонахождение обнаруженного на человеке металла. (Но они не смогут обнаружить пластиковую взрывчатку, подобную той, которая была обнаружена на «бомбардировщике в нижнем белье.”) См. Рисунок 5.

Рисунок 5 Знакомые ворота безопасности в аэропорту могут не только обнаруживать металлы, но и указывать их приблизительную высоту над полом. (Источник: Alexbuirds, Wikimedia Commons) Индуктивность

| электроника | Britannica

индуктивность , свойство проводника (часто в форме катушки), которое измеряется величиной электродвижущей силы или напряжения, индуцированного в нем, по сравнению со скоростью изменения электрического тока, который производит напряжение.Постоянный ток создает стационарное магнитное поле; Постоянно изменяющийся ток, переменный ток или флуктуирующий постоянный ток создают изменяющееся магнитное поле, которое, в свою очередь, индуцирует электродвижущую силу в проводнике, присутствующем в поле. Величина наведенной электродвижущей силы пропорциональна скорости изменения электрического тока. Коэффициент пропорциональности называется индуктивностью и определяется как значение электродвижущей силы, индуцированной в проводнике, деленное на величину скорости изменения тока, вызывающего индукцию.

Если электродвижущая сила индуцируется в проводнике, отличном от того, в котором изменяется ток, это явление называется взаимной индукцией, примером которой является трансформатор. Однако изменяющееся магнитное поле, вызванное изменяющимся током в проводнике, также индуцирует электродвижущую силу в самом проводнике, по которому протекает изменяющийся ток. Такое явление называется самоиндукцией, и отношение индуцированной электродвижущей силы к скорости изменения тока определяется как самоиндукция.

Британская викторина

Электричество: короткое замыкание и постоянный ток

В чем разница между электрическим проводником и изолятором? Кто придумал батарею? Почувствуйте, как ваши клетки горят, когда вы заряжаете свою умственную батарею, отвечая на вопросы этой викторины.

Самоиндуцированная электродвижущая сила противодействует изменению, которое ее вызывает.Следовательно, когда ток начинает течь через катушку с проволокой, он встречает сопротивление своему течению в дополнение к сопротивлению металлической проволоки. С другой стороны, когда электрическая цепь, несущая постоянный ток и содержащая катушку, внезапно размыкается, схлопывающееся и, следовательно, уменьшающееся магнитное поле вызывает индуцированную электродвижущую силу, которая стремится поддерживать ток и магнитное поле и может вызвать искру. между контактами переключателя. Таким образом, самоиндукцию катушки или просто ее индуктивность можно рассматривать как электромагнитную инерцию, свойство, которое противодействует изменениям как токов, так и магнитных полей.

Индуктивность зависит от размера и формы данного проводника, количества витков, если это катушка, и типа материала рядом с проводником. Катушка, намотанная на сердечник из мягкого железа, гораздо более эффективно подавляет увеличение тока, чем такая же катушка с воздушным сердечником. Железный сердечник увеличивает индуктивность; при той же скорости изменения тока в катушке, большая противодействующая электродвижущая сила (противо-ЭДС) присутствует, чтобы подавить ток.

Получите подписку Britannica Premium и получите доступ к эксклюзивному контенту.Подпишитесь сейчас

Единица магнитной индукции – генри, названный в честь американского физика 19 века Джозефа Генри, который первым открыл явление самоиндукции. Один генри эквивалентен одному вольту, разделенному на один ампер в секунду. Если ток, изменяющийся со скоростью один ампер в секунду, вызывает электродвижущую силу в один вольт, цепь имеет индуктивность в один генри, то есть относительно большую индуктивность.

САМОИНДУКЦИЯ-САМОИНДУКЦИЯ-ГЕНРИ

САМОСТОЯТЕЛЬНАЯ ИНДУКЦИЯ

Собственная индукция – это явление, при котором изменение электрического тока в Катушка создает наведенную ЭДС в самой катушке.
МАТЕМАТИЧЕСКАЯ ПРЕДСТАВИТЕЛЬСТВО:
Собственная наведенная ЭДС в катушке прямо пропорциональна скорости изменения электрический ток в катушке.

ЭДС а DI / Dt

Или ЭДС = -L DI / Dt

Где, L = собственная индуктивность катушки.

САМОСТОЯТЕЛЬНАЯ ИНДУКТИВНОСТЬ
Собственная индуктивность катушки определяется как отношение самоиндуцированной ЭДС к скорость изменения тока в катушке.

Я индуктивность = ЭДС / DI / Dt

Обозначается на L, и это зависит от физических характеристик катушки.

Единица себя индуктивность – Генри.

ГЕНРИ
Для последняя информация, бесплатные компьютерные курсы и важные заметки посетите: www.citycollegiate.com

Я индуктивность катушки составляет один генри, если изменение тока на один ампер на второй через него вырабатывает в нем ЭДС в один вольт.

1 генри = 1 вольт / 1 ампер / сек

ПОЯСНЕНИЕ САМОИНДУКЦИИ

Рассмотреть катушка, подключенная к батарее через реостат, как показано на рисунке.В ток через катушку создает магнитный поток, который связывается с сама катушка. Если мы изменим сопротивление в цепи, ток через катушка изменяется, и магнитный поток, проходящий через катушку, также изменяется. Этот изменение магнитного потока указывает на наличие ЭДС в самой катушке. Такая ЭДС называется самоиндуцированная ЭДС и это явление называется самоиндукцией.

Для последняя информация, бесплатные компьютерные курсы и важные заметки посетите: www.citycollegiate.com

Закон Фарадея

Закон Фарадея

Закон Фарадея

Закон Фарадея – одно из уравнений Максвелла. Закон Фарадея гласит, что абсолютная величина или величина обращения электрическое поле E вокруг замкнутого контура равно скорости изменения магнитный поток через область, ограниченную петлей. В приведенное ниже уравнение выражает закон Фарадея в математической форме.

ΔΦ B / Δt (через фиксированная площадь) = -Σ вокруг контура E ∙ r (при фиксированное время)

Знак минус в этом уравнении говорит нам о направлении тираж. (См. Ниже.)

Когда магнитный поток через замкнутую область при изменении петли Σ вокруг петли E ∙ r не равно нулю, электрическое поле E циркулирует.
E ∙
r – работа, выполненная за единичный заряд электрическим полем при перемещении заряда на расстояние ∆ r .
Если петля – это настоящая проволочная петля, тогда есть фактическая работа, выполняемая индуцированной поле на бесплатные начисления.
Σ вокруг петли E ∙ r – работа на единицу заряда полем при однократном перемещении заряда по петле.
Это наведенная ЭДС , и измеряется в вольтах.
Индуцированная ЭДС вызывает протекание тока без разность потенциалов из-за разделенных зарядов.

ΔΦ B / Δt (через фиксированная площадь) = наведенная ЭДС

Индуцированное электрическое поле НЕ консервативное поле. Когда вы перемещаете заряд против индуцированного поле один раз по кругу, вам нужно работать. Но твоя работа НЕ хранится как потенциальная энергия. Вы не можете позволить электрическому полю работать, чтобы восстановиться энергия, которую вы потратили на перемещение заряда.Индуцированное электрическое поле исчезает как как только магнитный поток перестанет меняться. Работа, которую вы делаете на заряд против индуцированного поля не локально хранится. Энергия может быть отведена в виде электромагнитная волна. Электромагнитные волны переносят энергию через свободное пространство.

Какое направление динамического (индуцированного) поля?

Знак минус в уравнении, выражающем закон Фарадея, говорит нам о направление индуцированного поля.
Есть простой способ запомнить это направление. Циркуляция индуцированного поля равна ЭДС.
Любой текущий течет в результате этой ЭДС создает магнитное поле, которое противодействует изменения потока, которые его производят.
Это называется Закон Ленца.

Индуцированная ЭДС действует как противодействие изменению потока, которое произвести это.

Пример:

Магнит быстро перемещается к проволочной петле, как показано.
Поток через проволочную петлю увеличивается в направлении вниз.
Ток начинает течь в петлю в направлении, указанном стрелкой.
Магнитное поле, создаваемое этим током указывает вверх, противостоит потоку изменения, которые его производят.
Магнитная сила из-за петли на магните действует, чтобы замедлить приближающийся магнит.

Прелесть закона Ленца заключается в том, что вам не нужно вдаваться в подробности.Если магнитный поток через проводник изменяется, токи будут течь встречно что бы ни вызвало изменение. Если какое-то относительное движение вызывает изменение потока, ток попытается остановить это относительное движение. Если изменение тока в цепь отвечает за изменение потока, тогда наведенная ЭДС будет пытаться предотвратить изменение тока в этой цепи.

Пожалуйста, смотрите: Электромагнитная индукция и закон Фарадея (Youtube)

Проблема:

Рассмотрим плоскую квадратную катушку с N = 5 витками.
Катушка имеет длину 20 см с каждой стороны и имеет магнитное поле. 0,3 Тл.
Плоскость катушки перпендикулярна плоскости магнитное поле: поле указывает за пределы страницы.
(a) Если ничего не изменилось, какова наведенная ЭДС?
(b) Магнитное поле равномерно увеличивается от 0,3 Тл до 0,8 Тл за 1 с. Какова наведенная ЭДС в катушке, пока происходит изменение?
(c) При изменении магнитного поля ЭДС, индуцированная в катушке, вызывает ток течь.Ток течет по часовой стрелке или против часовой стрелки? вокруг катушки?

Решение:

  • Рассуждение:
    Если величина магнитного поля B изменяется, то поток Φ = BA изменяется, и возникает ЭДС.
  • Детали расчета:
    (a) ЭДС индуцируется изменяющимся магнитным потоком. Если ничего изменяется, наведенная ЭДС равна нулю.
    (б) Катушка имеет 5 витков. Каждый поворот имеет площадь A = (0,2 м) 2 .Начальный магнитный поток через каждый оборот катушки Φ 0 = B 0 A = 0,3 * (0,2) 2 Tm 2 = 0,012 Tm 2 .
    Конечный магнитный поток через каждый виток катушки Φ f = B f A = 0,8 * (0,2) 2 Tm 2 = 0,032 Tm 2 .
    Суммарное изменение потока через катушку N (Φ ф – Φ 0 ), с N = 5. Индуцированная ЭДС равна
    ЭДС = -N∆Φ / ∆t = -N (Φ f – Φ 0 ) / ∆t = [-5 * (0.032 -0,012) / 1,0] V = -0,1 В.
    (в) При изменении магнитного поля магнитный поток увеличивался. со страницы. По закону Ленца наведенная в петле ЭДС этим изменяющимся потоком образуется ток, который создает поле, противодействующее изменение. Поле, создаваемое током в катушке, указывает на страницу, противоположную направлению увеличения потока. Чтобы произвести поле на страницу, ток должен течь по часовой стрелке по петле согласно правилу правой руки.

Модуль 5: Вопрос 1

Стержневой магнит расположен перед горизонтальной проволочной петлей с его северный полюс, указывающий на петлю. Затем магнит отрывается от петля. Идет ли индуцированный ток в контуре по часовой стрелке или против часовой стрелки?

Обсудите это со своими однокурсниками на дискуссионном форуме!
Визуализируйте магнитное поле стержневого магнита. Как происходит поток этого поле через проводную петлю поменять?


Самоиндукция

Если длинная катушка провода сечением A и длиной ℓ с N витками подключен или отключен от батареи, изменение магнитного потока через катушка производит наведенную ЭДС.Индуцированный ток создает магнитное поле, которое противодействует изменению магнитного потока. Величина наведенная ЭДС может быть рассчитана с помощью закона Фарадея.

  • Магнитное поле внутри длинной катушки B = μ 0 (N / ℓ) I.
  • Поток через катушку равен NBA = μ 0 (N 2 /) IA.
  • Изменение потока в единицу времени составляет μ 0 (N 2 / ℓ) A ∆I / ∆t = L * ∆I / ∆t, поскольку I – единственная величина меняется со временем.
    L = μ 0 (N 2 / ℓ) A называется собственная индуктивность катушки. В единицы индуктивности – Генри (Гн) . 1 H = 1 Вс / А.
  • Индуцированная ЭДС равна ЭДС = -L * ∆I / ∆t, где знак минус является следствием закона Ленца.

Индуцированная ЭДС пропорциональна скорости изменения тока в катушка. Оно может быть в несколько раз больше напряжения источника питания. Когда выключатель в цепи, по которой проходит большой ток, размыкается, уменьшая ток до ноль за очень короткий промежуток времени, это может привести к искре.Все схемы имеют собственную индуктивность, и у нас всегда есть ЭДС = -L * ∆I / ∆t. Собственная индуктивность L зависит только от по геометрии схемы.

проблема:
Катушка

А имеет собственную индуктивность 3 мГн, а ток через нее изменяется от 0,2 А. до 1,5 А за время 0,2 с. Найти величину средней наведенной ЭДС в катушке за это время.

Решение:

  • Рассуждение:
    ЭДС самоиндукции равна ЭДС = -L * ∆I / ∆t.
  • Детали расчета:
    L = 3 мГн, ∆I / ∆t = (1,5 A – 0,2 A) / 0,2 с = 6,5 A / с.
    э. Д. произвел это.
Проблема:

Круглая катушка с 25 витками проволоки диаметром 1 м. Он размещен со своим ось вдоль направления магнитного поля Земли (величина 50 мкТл), а затем в 0.2 с переворачивается 180 o . Какая средняя ЭДС сгенерировано

Решение:

  • Рассуждение:
    Φ B = B A – поток B через область A. Первоначально B и A выровнены, наконец, они анти-выровнены. Точка товар меняет знак.
  • Детали расчета:
    ЭДС = -∆Φ B / ∆t. Φ B (начальная) = NAB = 25 * π * (0,5 м) 2 50 * 10 -6 Т = 9.82 * 10 -4 Тм 2 .
    Φ B (окончательный) = -Φ B (начальный), поскольку катушка перевернута.
    | ∆Φ B | = 2Φ B (начальное).
    | ∆Φ B / ∆t | знак равно 2 * (9,82 * 10 -4 Tm 2 ) / (0,2 с) = 9,82 * 10 -3 В.
Проблема:

Катушка с 500 витками радиусом 0,5 м поворачивается на четверть оборота за 4,17. мс, изначально имеющая плоскость, перпендикулярную однородному магнитному полю. Найдите напряженность магнитного поля, необходимую для индукции средней ЭДС 10 000 В.

Решение:

  • Рассуждение:
    ЭДС = -∆Φ B / ∆t. Φ B = NABcosθ изменяется с NAB на 0 за 4,17 мс, так как θ изменяется от 0 до 90 o через 4,17 мс.
  • Детали расчета:
    | ∆Φ B | = NAB = 500 * π * (0,5 м) 2 * B = (393 м 2 ) * Б.
    Мы хотим
    | emf | = | ∆Φ B / ∆t | = (393 м 2 ) / (4.17 * 10 -3 с) * B = (94174 м 2 / с) * B = 10000 В.
    B = 0,1 Вс / м 2 = 0,1 Т.

Если вы пропускаете регулярные лекции, обратите внимание на эту видеолекцию.

Лекция 16: Электромагнитная индукция

Самоиндуктивность

– определение, формула, единицы измерения и часто задаваемые вопросы

Давайте рассмотрим волшебство концепции самоиндукции.

Рассмотрим катушку и пропустим через нее ток, не только ток, но и изменяющийся ток.

Теперь из-за изменяющегося тока в нем индуцируется дополнительный ток, то есть индуцированный ток.

Итак, вы знаете, что означает этот наведенный ток?

Ну, индуцированный ток нематериален, и это свойство катушки генерировать его из-за изменяющегося тока, подаваемого нами через батарею.

Не все ли так просто?

Однако наше внимание уделяется самоиндукции, поэтому давайте вернемся к рассмотрению этой волшебной концепции.

Итак, что вы наблюдали в этом явлении и почему оно волшебное?

Итак, магия заключается в следующем: при передаче тока в катушку она индуцирует внутри себя ток, известный как самоиндуцированный.Вот почему это явление известно как самоиндукция.

Собственная индуктивность катушки

Рассмотрим катушку и пропустим через нее ток. Подача тока является первичным током, и здесь мы рассмотрим два случая, а именно:

  1. Подача, которая продолжает увеличиваться, и

  2. Подача тока уменьшается.

Случай a: Рассмотрим катушку, в которой первичный (подаваемый) ток увеличивается в направлении, показанном ниже на диаграмме.

(изображение будет загружено в ближайшее время)

Как вы знаете, возрастающий (изменяющийся) ток сам по себе генерирует индуцированный ток, то есть самоиндуцированный ток, но он течет в направлении, противоположном направлению тока ток поставлен. Направление этого самоиндуцированного тока показано на диаграмме ниже.

(изображение будет загружено в ближайшее время)

Этот индуцированный ток препятствует любому изменению (или увеличению тока) того тока, из-за которого он возникает.

Теперь возьмем другой случай:

Случай b: Рассмотрим катушку, в которой ток уменьшается. Здесь происходит то, что индуцированный ток способствует изменению (или уменьшению) приложенного тока.

Это означает, что индуцированный ток течет в направлении приложенного тока и способствует его увеличению.

(изображение будет загружено в ближайшее время)

Мы поняли, что индуцированный ток препятствует увеличению тока и поддерживает уменьшение тока.

(изображение скоро будет загружено)

Что такое самоиндуктивность?

Рассмотрим круг, в котором изменяющийся ток создает магнитное поле (B).

Направление этого поля можно определить, согнув пальцы правой руки, и мы получим направление B, которое указывает внутрь, что можно увидеть в виде крестиков на диаграмме ниже:

(изображение будет загружу скоро)

Теперь при увеличении тока силовые линии магнитного поля тоже увеличиваются. Это означает B α i.

Из-за увеличения B также увеличивается поток (ΦB).

Как только поток увеличивается, тогда, согласно закону индукции Фарадея, в этой катушке возникает наведенная ЭДС.

По закону Ленца,

Эта наведенная ЭДС представляет собой разность потенциалов между двумя точками этой катушки, из-за которой генерируется наведенный ток. Этот индуцированный ток уменьшит первичный ток. Его направление наружу, то есть противоположно направлению B.

Этот ток создает свой поток, который противодействует потоку (ΦB), из-за которого он был создан.

Итак, это явление самоиндукции.

Формула самоиндуктивности

Давайте рассмотрим катушку с током, имеющую количество витков N, как показано ниже:

(изображение будет загружено в ближайшее время)

Если поток через одну катушку равен Φ, то для N количества витков катушек, это будет:

ΦT (общий поток) = NΦ, и

ΦT α i

Убрав знак пропорциональности, получим

ΦT = Li ⇒ L = \ [\ frac {N \ Phi T} {i} \]

Где L – коэффициент самоиндукции.

Здесь L – постоянная, не зависящая от Φ и i. Скорее, это зависит от следующего:

  1. Геометрия,

  2. Форма и

  3. Размер индуктора (катушки).

Это означает, что индуктивность не изменяется при увеличении или уменьшении при изменении Φ и i.

Единица самоиндукции

Единица самоиндукции – Вебер / Ампер или Генри «H».

Размер самоиндукции составляет [ML 2 T -2 A -2 ].

Определение коэффициента самоиндукции

По закону Фарадея ЭДС, \ [e = – \ frac {\ Phi T} {dt} \]

Итак, \ [e = | – L \ frac {di} {dt} | \ Rightarrow L = \ frac {e} {| \ frac {di} {dt} |} \]

Если значение изменения тока или di / dt составляет 1 ампер / секунду, тогда L = e. Это определение коэффициента самоиндукции.

Мы знаем, что индуктивность – это свойство электрического проводника, благодаря которому изменение тока вызывает ЭДС.

Самоиндукция и взаимная индукция

S.№

Самоиндукция

Взаимная индукция

1.

Добавить комментарий

Ваш адрес email не будет опубликован. Обязательные поля помечены *